Наука

Ответить в тред Ответить в тред
Check this out!
Тред тупых вопросов Прошлый >>514446 (OP) Тред тупых вопросов 13/10/20 Втр 19:06:13 5177981
Screenshot21.png 548Кб, 630x519
630x519
Тред тупых вопросов

Прошлый >>514446 (OP)
Аноним 13/10/20 Втр 19:12:23 5177992
16025203597470.mp4 13777Кб, 1280x720, 00:01:29
1280x720
image.png 662Кб, 600x708
600x708
Автомат создающий неведомую хуйню. Меня интересует нагромождение конструкций справа, по-сути тут нужно зафиксировать проволоку двумя упорами, зачем этим упорам уделили столько внимания и конструкторских ресурсов. Понятно что их надо подогнать до тысячной доли миллиметра, но все же. У меня только одна идея, техника следит за температурой и если в течении дня она меняется, то упоры реагируют на это.
Аноним 13/10/20 Втр 19:15:01 5178003
Поясните за криптовалюту, деньги создаются из воздуха? Кому и зачем это нужно
Аноним 13/10/20 Втр 20:23:54 5178174
>>517800
Деньги это что большинство людей приняли и используют для оценки цены чего-либо. Деньгами может быть все что угодно, но сейчас отошли к полностью абстрактным величинам. И да, криптовалюты не являются ни деньгами, ни платежным средством и никогда ими не станут..
Криптовалюты по своей сути это решение уравнений, со специальными свойствами. Всякие маняфантазеры решели задать цену этим решениям и начали покупать и торговать ими. Цену по идеи создают спрос и предложение, однако из-за своей особенности криптовалюты быстро не могут достигнуть равновесии цены. В итоге криптовалюты превратились в спекулятивный актив/финансовую пирамиду. И имеем то, что имеем.
Люди не создают деньги, они просто продают другим нечто, что хорошо покупают. Однако приход новых игроков рано и поздно закончится(люди не бесконечные) и рынок криптовалюты выродится в обычный рынок акций.
Аноним 13/10/20 Втр 20:37:53 5178205
>>517817
Ну хотя бы крипта как-то развивает технологии, специальные видяхи для них пилят.
Аноним 13/10/20 Втр 20:44:23 5178216
Untitled-2.jpg 44Кб, 367x476
367x476
>>517799
>Понятно что их надо подогнать до тысячной доли миллиметра
Понятно? Мне, например, не понятно. Нахуя микронная точность в пружине, которая по самой своей сути предназначена для того, чтобы деформироваться и быть деформированной?

К тому же там рядом какие-то сантиметровые шкалы стоят. Могу предположить, что вся эта йоба нужна для универсальности, чтобы можно было настраивать разные диаметры и шаги витков у пружин.
Аноним 13/10/20 Втр 20:46:16 5178247
>>517821
Или может просто создали бутафорию и продали по дороже, типа производственный сговор?
Аноним 13/10/20 Втр 21:36:45 5178328
В соседнем треде позвучал вопрос какой вид бактерий самый многочисленный. Если тупо загуглить, то выдает самую крупную бактерию. Так какая же? Вангую та что живет в почве
Аноним 13/10/20 Втр 21:56:03 5178379
>>517832
Ты на океан внимательно посмотри, он буквально из себя представляет солёный супчик из всей цепочки от аминокислот до млекопитающих.
Аноним 13/10/20 Втр 21:56:04 51783810
Аноним 13/10/20 Втр 22:11:22 51784011
Тогда еще вопрос по бактериям, у них есть какой-то лимит на размер как у насекомых и можно ли методом селекции и генной инженерии создать гигантскую бактерию?
Заранее предполагаю что она будет сладкой булочкой для окружающих микроорганизмов и животных питающихся мелочевкой.
Аноним 14/10/20 Срд 02:42:12 51785912
>>517840
Физический размер клетки ограничивается поверхностным натяжением мембран. Можно собрать почти миллиметровую клетку, но для этого потребуется суперпрочное усиление внешней мембраны - по сути строиться внешний скелет. Энергетическая цена такого скелета для одной клетки слишком высокая. Впрочем по этому пути пошли многоклеточные через специализацию. Бактерии же избрали другой путь - максимальной энергетической эффективности. Пожертвовав большим размером и запасами, они в замен получили самые эффективные ферменты и скоростью адаптации. Размер бактерий определяется питательным субстратом и плотностью колонии. Конечно можно вывести бактерий гигантов, но попав в обычную среду, они быстро вернутся к обычным размером.
Аноним 14/10/20 Срд 05:06:12 51786113
Если взять современного олигофрена в стадии дебильности (легкое слабоумие) и отправить в прошлое, например в каменный век, сможет ли он занять достойное место и вести привычный для той среды образ жизни.

Что будет если отправить его в средневековье.

Кто не совсем понимает как выглядит дебил, он способен вести несложные беседы, подтереть жопу после туалета и помыть полы шваброй за шоколадку. Но с письмом и математикой у него проблемы.
Аноним 14/10/20 Срд 05:31:57 51786214
>>517861
Люди всегда использовали свой мозг полностью, а не только на письмо и математику.
Такой большой "запас" не появился бы просто так, никакого запаса не было.

Наоборот сейчас дегенератам гораздо легче, фактически вообще ничего не надо делать многим, просто приходить вовремя на работу, выполнять машинную работу как на конвеере, всё остальное, сложное сделает за тебя разделение труда. Транспорт ездит, батареи греют, вода из крана течёт, в магазинах еда, минимальную зп платят гарантированно практически, жильё от совка бесплатное или от социальной параши.
Аноним 14/10/20 Срд 05:59:52 51786415
>>517862
>Наоборот сейчас дегенератам гораздо легче, фактически вообще ничего не надо делать многим, просто приходить вовремя на работу, выполнять машинную работу как на конвеере, всё остальное, сложное сделает за тебя разделение труда.
Звучит как речь РНН-господина. Не совсем понятно что ты имеешь в виду. Чтобы устроиться на не пыльную работу нужно нужно хотя бы складывать в уме трехзначные числа. Даже пьющие грузчик и дворник не совсем дегенераты, как минимум они должны снимать штаны прежде чем посрать.
Аноним 14/10/20 Срд 06:10:16 51786516
>>517864
>Чтобы устроиться на не пыльную работу
Ты спрашивал про ум, раньше тупые сдыхали, т.к. не было разделения труда, нужно было знать и уметь ооооочень много вещей.
Никто не говорил что она будет непыльной. Она будет несложной умственно. При этом минимальную зп точно будут платить, а на неё легко можно выжить.
Аноним 14/10/20 Срд 06:20:04 51786617
>>517865
>При этом минимальную зп точно будут платить, а на неё легко можно выжить.
У тебя искаженные представления о реальности. Люди жопу рвут чтоб заработать ссаные копейки. При безработице просто хуй сосут, иногда в прямом смысле. От нищеты вешаются. Процент суицидов в этой стране чекни.
Аноним 14/10/20 Срд 06:27:22 51786718
>>517866
Хватит себя оправдывать и сидеть на мамкиной шее, все кто хотят, даже совсем конченные дегенераты-все работают и живут самостоятельно.
Аноним 14/10/20 Срд 06:28:30 51786819
>>517864
Умение складывать числа и читать не показатель интеллекта и ума. Большинство людей не могут осилить понятие предела.
Несколько сотен лет назад от человека для выживания требовались всякие безумные умения, которым он должен был быстро обучаться, причем сам без гайдов и подсказок. Попутно человек был сильно привязан к обществу, обросшим огромным количеством правил и условностей, нарушение которых сделало бы его изгоем, что сродни смерти.
Современная же среда позволяет человеку вполне выжить одному с минимальными количеством умений, а мораль общества не ограничивает его деспотичными обязательствами.
Аноним 14/10/20 Срд 06:37:07 51786920
>>517868
Ты Савельева наслушался что ли? Жизнь человека все время усложняется, но сложность поднимается с физической в умственную область. Если раньше человек должен был пахать поле на лошаденке или сажать рис вручную, то сейчас больше задумывается над своими действиями и над всякими абстрактными вопросами. Даже если хиккан никуда кроме школы не ходит и в классе не поддерживает контактов, в его башке постоянно крутятся внутренние диалоги и анализ своего и поведения и окружающих.
Аноним 14/10/20 Срд 06:42:29 51787021
>>517867
Боже, реально дегенераты живут среди нас. Я на минуту забыл, но твой пост заставил меня вспомнить реальность.
Аноним 14/10/20 Срд 08:29:44 51787122
>>517861
> в каменный век
Ему там пизда сразу.
>в средневековье
Может повезти выжить. Если сможет пристроиться в какой-то монастырь и будет ебашить от зари до заката, то ему будут давать кусок хлеба и воду.
Аноним 14/10/20 Срд 08:35:45 51787323
>>517871
Почему пизда. Ему не надо соперничать с людьми на порядок более развитых себя, не надо изучать письменность и упражняться в красноречии. Половина коммуникаций это жесты и рычание. Действия куда проще, берешь палку и копаешь, берешь камень и кидаешься в гигантского ленивца чтобы сбить его с ветки. Он будет как рыба в воде, может лет через пять сумеет добыть огонь, по воспоминаниям из нашего времени.
Аноним 14/10/20 Срд 09:31:04 51787724
>>517873
Никакой ум тебе не поможет, когда тебе могут вынести ебучку без вариантов.
Аноним 14/10/20 Срд 09:40:05 51788025
>>517877
А что мешает уебать камнем в ответ. Наш дебил похитрее и завалит обидчика во время сна, да так что тот кровью истечет без шансов. Остальные будут бояться и уважать
Аноним 14/10/20 Срд 09:41:05 51788126
s1200.jpg 205Кб, 1200x916
1200x916
>>517798 (OP)
Есть ли доказательство наличия связи между причиной и следствием?
Аноним 14/10/20 Срд 10:14:06 51788627
Напомните, как называется эффект, когда, например, муж и жена оба по отдельности не хотят идти в кино, но оба идут, потому что каждый жертвует своей хотелкой, думая, что его половинка хочет идти.
Аноним 14/10/20 Срд 10:25:46 51788928
>>517869
>пахать поле на лошаденке или сажать рис вручную
А потом твоя лошаденка внезапно запаршивила и вдруг выяснилось, что для того чтобы пахать поле ты оказывается должен уметь ухаживать за лошаденкой, а не просто пахать. А когда твой рис нихуя не взошел, перед тобой ненавязчиво возник выбор - быстро понять что с ним не так, сдохнуть с голоду или попытаться придумать чем заменить пропавший рис. И так во всем.
Аноним 14/10/20 Срд 10:28:57 51789029
>>517880
>дебил похитрее
Проиграл с дебила, который вообразил, что он хитрее нормисов потому что знает как спрятать сухарь, чтобы санитар не нашел
Аноним 14/10/20 Срд 14:58:53 51789530
>>517861
Нет конечно. Тогда не было нужного развития НТР и нужно было дохуя всего уметь делать. Мозг человека уменьшается последние несколько тысяч лет, поскольку жить становится легче. Представь каменный век, это пиздец сколько нужно навыков чтобы просто не сдохнуть. Ты должен быстро уметь ориентироваться на местности, уметь в анализ растений и животных, уметь вести охоту на много разной дичи, уметь делать одежду, уметь точить и делать оружие из подручных материалов (напомню, веревок и ножей нет). Понимать как лечить себя в случае пиздеца и всё это с минимальными научными знаниями и чисто на интуитивном уровне. Ты должен был обладать максимальной обучаемостью, чтобы просто жить.
Аноним 14/10/20 Срд 15:00:04 51789631
>>517881
Если в широком философском смысле, то его нет.
Аноним 14/10/20 Срд 16:05:07 51790132
>>517889
Ты себе плохо представляешь сколько крестьян сдохли от голода
Аноним 14/10/20 Срд 16:06:47 51790233
>>517890
Так те нормисы были уровня нашего дебила, но наш дебил повидал цивилизацию и знает поболее
Аноним 14/10/20 Срд 16:11:52 51790334
>>517895
>Мозг человека уменьшается последние несколько тысяч лет, поскольку жить становится легче
Сычуш, опять научпока насмотрелся. А компьютеры и телефоны измельчали потому что слабее стали. А может мозг стал легче чтоб снять часть нагрузки на позвоночник и в целом стал более эноргономичным, то есть потребляет меньше энергии при большей производительности. Я бы пошутил про мозг слонов, но мне лень.
Аноним 14/10/20 Срд 16:19:15 51790435
>>517901
Это ты к чему сейчас? Опят про то, что даун умнее крестьянина?
Я начинаю подозревать, что ты и есть тот даун о котором тут речь.
Аноним 14/10/20 Срд 16:22:26 51790536
>>517902
>наш дебил повидал цивилизацию и знает поболее
Поболее он знает про то, как ездить в метро, лифте и покупать хлеб в магазине. Ни одно из этих знаний в средневековье ему не пригодится.
Аноним 14/10/20 Срд 16:53:27 51790737
Александр Марко[...].mp4 14088Кб, 640x360, 00:05:25
640x360
>>517903
Антропологи этот вариант рассматривали. Быстро отмели. Отделы мозга, отвечающие за многие вещи деградируют. Да что там говорить, сейчас даже самые простые вещи стали деградировать.
Аноним 14/10/20 Срд 18:45:53 51791438
>>517904
>Опят про то, что даун умнее крестьянина
Что значит опят если я этого не говорил. Даун средневековья тупее чем даун 21 века.
Аноним 14/10/20 Срд 18:48:04 51791539
>>517914
Даун, естественный отбор даёт тебе на рота.
Аноним 14/10/20 Срд 18:51:57 51791640
>>517907
То есть что-то упрощается, а что-то усложняется? У того дикаря с огромным мозгов темени не было, в сравнении с современником. Зрение слух обоняние лучше понятное дело, животные и охотничьи инстинкты преобладали. Соответствующие области мозга развиты под это дело. Ты еще мозг кита не видел, который в основном отвечает за слух и в котором постоянно отмирают клетки из-за кислородного голодания. А поведение не сильно отличается от дельфиньего, чей мозг заметно меньше. Почему-то размеро-дауны до сих пор не вымерли, видимо их плохо обоссывают.
Аноним 14/10/20 Срд 18:53:33 51791741
16006524123540.png 4Кб, 170x127
170x127
Аноним 14/10/20 Срд 19:00:34 51791842
>>517917
Даун, ты чего порвался?
Аноним 14/10/20 Срд 19:08:36 51792043
>>517918
бля, посты малолетнего долбоеба не скрываются, хохлоскрипт не робит, страница бесконечно загружается. попробовал на оперу скрипт накатить, нихуя не выходит. ливаю отсюда пока баг не починят. Спасибо, Абу и мочератор, что у вас 13тилетние пиздюки тут сидят
Аноним 14/10/20 Срд 19:27:09 51792144
Что принципиально плохого в инцесте? Отбросим возможные генетически последствия инцестных детей, т.к. подсознание этого не учитывает, но зато ребёнка подсознательно влечёт к своей матери. Я предполагаю что проблема в том, что между отцом и сыном может возникнуть конфликт за вожделеющий сексуальный объект в качестве матери. А если отец не против, что его сын будет поёбывать его мать, а мать готова удовлетворить сына? Вот отец договорился с сыном, что, мол, будешь хорошо учиться - мать будет тебе дрочить и ты сможешь её гладить что угодно. Может ли что-то катастрофичное произойти в принципе?

мимо просто наткнулся на эдипов комплекс и решил прояснить для себя этот момент
Аноним 14/10/20 Срд 19:54:27 51792345
>>517920
Опущенная порватка даунская, зашивайся.
Аноним 14/10/20 Срд 19:54:54 51792446
>>517921
Пиздуй в ph/, дегенерат.
Аноним 15/10/20 Чтв 03:03:22 51795847
>>517921
При инцесте у будущих поколений снижается интеллект, если посмотреть на американских хилли билли. Также замедляется или останавливается эволюция, в частности, у потомков будет ослабленный иммунитет, при том, что вирусы будут продолжать эволюционировать.
Чтобы потомство не вырождалось, эволюционно закрепляется отвращение к инцесту на генетическом уровне. Это распространяется даже на ближайшее окружение, людей, с которыми человек рос с детства - соседи и др. Нормальный человек не может их воспринимать в качестве партнера для секса.
Про сексуальное влечение к родителям - это антинаучный бред Фрейда, ничем не подтвержденный. Психоанализ не фальсифицируем, следовательно к науке имеет такое же отношение, как креационистская теория.
Аноним 15/10/20 Чтв 04:14:45 51795948
>>517886
Парадокс Абилина
Аноним 15/10/20 Чтв 05:04:51 51796049
бонс.jpg 38Кб, 480x360
480x360
Я не понимаю с какой стати может идти речь о каком-то большом отскоке.

Типа вселенная это как камень подброшенный вверх, на какое-то время он оторвался от земли, но гравитация всё равно победит и через какое-то время он опять ёбнется на землю.

Во-первых это же противоречит наблюдаемому. Скорость расширения вселенной даже не думает замедляться. Скорее наоборот.
Ладно, представим что не противоречит. Представим что всё обратно стягивается.
Но разве это не значит, что по итогу вся материя просто сожмётся в одну большую чёрную дыру и на этом всё?
Аноним 15/10/20 Чтв 05:28:05 51796150
>>517960
Когда у тебя сжимается сама вселенная, черная дыра не конечный объект.
Аноним 15/10/20 Чтв 05:47:52 51796351
>>517961
Окей, представим что эта чёрная дыра по итогу, через необозримые эоны лет, испаряется из-за излучения Хокинга. Что дальше?
Аноним 15/10/20 Чтв 06:12:54 51796452
>>517963
Это не черная дыра.
Аноним 15/10/20 Чтв 12:33:38 51797153
Может ли в реальности существовать всё то, что можно помыслить, нафантазировать? Скажем, на алмазной планете отчим кенгуру в трениках потёртых насилует кошку на спине гигантской черепахи с именем Бастурма. Я это придумал в рамках нашей вселенной. Значит на одной из триллионов триллионов планет такое возможно?
Аноним 15/10/20 Чтв 13:01:15 51797254
>>517971
Чтобы это доказать или опровергнуть тебе нужно изучить всю вселенную, включая те области, что лежат за космологическим хорайзоном. Удачи.
Аноним 15/10/20 Чтв 16:53:20 51799155
>>517905
Средневековье наше прошлое, мы уже переросли ту социальную модель, стали старше. Дебил прикинется блаженным, научится плести религиозную ахинею, типа видел ангелов и с богом говорил, ему будут монеточку кидать, а он радоваться и кривляться. Короче он будет вертеть средневековое было на хую
Аноним 15/10/20 Чтв 16:55:03 51799256
>>517923
Всё, дебил, я починил, ори дальше пока модерок тебя не кикнет
Аноним 15/10/20 Чтв 17:38:54 51799957
>>517959
Ай ты лапа, спасибище!
Аноним 15/10/20 Чтв 18:31:51 51800258
>>517971
Если вселенная бесконечна, то возможно абсолютно всё, даже присутствие или отсутствие того что ты описал.
Аноним 15/10/20 Чтв 18:33:46 51800359
>>518002
>вселенная бесконечна
Нет. Масса не бесконечна, иначе все схлопнулось в кашачий сфинктер
Аноним 15/10/20 Чтв 18:57:47 51800660
>>518003
Ваши доказательства?
И масса схлопывается к центру масс, а в бесконечности нет центра.
Аноним 15/10/20 Чтв 19:06:58 51800961
>>518006
Боже, какая к хуям бесконечность. Ночное небо светило звездами как днем если рассчитать не капая глубоко. А если копнуть, то любая бесконечность поглотит всё вообще, не оставив места ничему.
Аноним 15/10/20 Чтв 19:13:22 51801062
>>518009
что ты несёшь поехавший штоле?
Аноним 15/10/20 Чтв 19:17:46 51801163
>>518002
Ты хоть понимаешь, что эту хуету форсят веруны, потому что из нее выходит, что в бесконечной вселенной достоверно возможно существование бога.
А по факту
1) никаких доказательств бесконечности вселенной нет, скорее наоборот
2) даже в бесконечной вселенной, если вечный игральный кубик с цифрами от 1 до 6, бросать бесконечно долго, то сколько его ни бросай 7 на нем не выпадет никогда.
Шах и мат.
Аноним 15/10/20 Чтв 19:25:39 51801264
>>518010
Ты слишком туп. Я предложу проверку на бесконечность, которую можно совершить не вставая с дивана. Ты не поймешь, даже не отвечай, поймут другие. Берем любой предмет, яблоко, оно имеет контуры и конечные параметры, такие как масса и размер. И все предметы, включая астрономические объекты имеет конечную массу и размер. Теперь размышляем о времени, мы знаем твой возраст, лет 16, и средний срок жизни человека пусть 70 лет, есть начало и конец. Даже у вселенной есть начала отсчета. Следовательно, всё имеет начало и конец. Тут уже философия вмешивается в физику, но это неизбежно. Если ты думаешь что объекты с конечной массой образуют бесконечность, то земля тебе пухом. Бесконечность обладает свойствами с которыми мы не сталкиваемся в реальности и эти свойства вступают в кофликт с наблюдаемым миром. Бесконечность это для верунов, у которых бох был всегда и бесконечно всех любит.
Аноним 15/10/20 Чтв 19:27:20 51801365
>>518011
Бог возможен даже в конечной вселенной. Физика это не отрицает.
Аноним 15/10/20 Чтв 19:45:55 51801666
>>518011
>никаких доказательств бесконечности вселенной нет, скорее наоборот
Приведи эти доказательства.
>вечный игральный кубик с цифрами от 1 до 6, бросать бесконечно долго, то сколько его ни бросай 7 на нем не выпадет никогда
А я говорю выпадет. И чо?
>>518012
>всё имеет начало и конец
И как это отменяет бесконечность?
И самое главное:
Ты говоришь что бога нет, но при этом вселенная у тебя появилась сама по себе из ничего, чудесным образом.



Аноним 15/10/20 Чтв 19:50:11 51801767
Аноним 15/10/20 Чтв 19:51:56 51801968
>>518016
>Ты говоришь что бога нет,
Ты с Пикабу? Я этого нигде не говорю. Или ты про фразу
>>518009
>Боже, какая к хуям бесконечность

>>518016
>при этом вселенная у тебя появилась сама по себе из ничего, чудесным образом
Ну это проблема дырявой теории большого взрыва, а не моя.
Аноним 15/10/20 Чтв 19:52:29 51802069
>>518017
Ну репортни его, борда 18+
Аноним 15/10/20 Чтв 20:32:32 51802170
>>518012
>конечные контуры и параметры
А точно ли? С чего ты уверен, что поверхность яблока - не представлена бесконечно малой самоподобной фигурой? Тогда периметр его вовсе не конечен. Ты путаешь понятия конечности, исчислимой и неисчислимой бесконечностей.
Аноним 15/10/20 Чтв 21:01:55 51802271
>>518016
>ВРЕТИЕ
>выпадет
найс ты подгорел, верунок
Аноним 15/10/20 Чтв 21:18:30 51802472
>>517964
Почему?
Материю разнесло во все стороны. Потом её сжимает обратно. Когда материю сильно сжимает, она становится чёрной дырой. Разве нет?
Аноним 15/10/20 Чтв 21:19:31 51802573
>>518021
А нахуя ты математику пытаешься натянуть на реальность. Яблоко конечно, это даже дебил понимает, но не шизик с двачей. Циферки лишь у тебя в голове, их не существует в природе.
Аноним 15/10/20 Чтв 22:25:15 51802874
>>518025
Прикольно что это даже, хм, в обратную сторону работает.

Т.е. сейчас Пи вычелсенно точнее, чем существуют в природе самые идеальные сферы, даже если взять сферу, идеально ровную, но из реальных атомов, то она будет многоугольником.
Аноним 15/10/20 Чтв 22:38:10 51803075
image.png 559Кб, 700x612
700x612
>>518028
А толку. Возьми очень крупный объект, гигантскую звезду или галактику и примени 3,14 в ее расчетах. Где нибудь эта точность пригодиться
Аноним 15/10/20 Чтв 23:34:34 51803576
>>518030
Сейчас она вычислена гораздо точнее чем планковская длина по отношению к наблюдаемому радиусу вселенной.

Где пригодится?
Аноним 15/10/20 Чтв 23:36:00 51803677
Слушьте, а ведь фундаментальная константа должна быть только одна, а все остальные должны быть следствием из неё?
Аноним 16/10/20 Птн 00:24:26 51803978
>>518036
При про какую именно?
Аноним 16/10/20 Птн 00:41:26 51804079
>>518035
Когда будем строить виртуальную вселенную аля матрица
Аноним 16/10/20 Птн 00:55:46 51804180
>>518039
Вообще, её походу пока нету, открытой,
Может это одна из тех что уже есть, но остальные пока не завязали на основе её. А может это и какая-то совсем другая.
Аноним 16/10/20 Птн 01:52:21 51804481
>>518024
>>517960
Потому что большой взрыв не из одной точки а во всех точках сразу. Поэтому где у тебя сингулярность будет? Во всех точках? Если во все стороны тянет одинаково, то и черной дыры нет.
Аноним 16/10/20 Птн 01:54:04 51804582
>>517820
>Ну хотя бы крипта как-то развивает технологии, специальные видяхи для них пилят.
Охуеть технологии, хэши считать. ля криптовалют делают айсики уже давно.
Аноним 16/10/20 Птн 02:00:37 51804683
>>517881
>Есть ли доказательство наличия связи между причиной и следствием?
Картина мира вне причинно-следственного принципа не имеет смысла. Ибо принципиально не позволяет создавать теории обладающие предсказательной силой.

>>517907
>Антропологи этот вариант рассматривали. Быстро отмели. Отделы мозга, отвечающие за многие вещи деградируют. Да что там говорить, сейчас даже самые простые вещи стали деградировать.
Раньше чем случится критическая деградация, человечество перейдёт к социализму, а затем и к коммунизму. А там уже половой отбор (в размножении) поправит интеллект популяции.
Аноним 16/10/20 Птн 02:18:24 51804784
>>518045
Я помню когда додумались вместо процессора видюхой вычислять протеины, то это был типа инновационный прорыв
Аноним 16/10/20 Птн 03:01:07 51804885
>>518046
Новая секта деградации мозга, забавно
Аноним 16/10/20 Птн 03:32:39 51804986
>>518044
Чёрные дыры, если они находятся достаточно близко, сливаются в одну. Так ведь и запруфали гравитационные волны. Зарегистрировав слияние двух чёрных дыр.
Поэтому [по итогу] сингулярность будет только в одной точке. И совершенно непонятно, как из этой, ооочень медленно испаряющейся чёрной дыры, должна бы вновь возникнуть вселенная.
Аноним 16/10/20 Птн 03:42:16 51805087
>>518049
Блядь, ебанат тупоголовый, вселенная не сожмется в черную дыру, а в то, что там было до инфляции. Черти ебаные, черные дыры это другое.
Аноним 16/10/20 Птн 03:48:28 51805188
>>518050
Нет, погоди.
Материю разметало в разные стороны? Разметало.
Когда гравитация сильно сжимаает материю, она [материя] становится чёрной дырой? Становится.
Так с хуя какого ты блядь бомбишь, уёбище блядь? И внятно блядь, сука, внятно, по-человечески поясняй. Если же не можешь, если не в состоянии, если мощностей не хватает то и хавальник свой не раззевай попусту.
Аноним 16/10/20 Птн 03:53:39 51805289
>>518051
Какая материя, пидорас ебаный? У тебя, залупоголового трюфеля, сжимается сама ткань вселенной, а ты свою разъебаную черную дырку приплетаешь.
Аноним 16/10/20 Птн 03:58:52 51805390
>>518052
Какая нахуй "ткань"? Какая "ткань" нахуй, ебаната кусок? Что ты блядь несёшь, припиздок блядь ты ёбанный, а?
Ты разберись хоть в чём-нибудь сначала, отвечать он тут сука взялся. "Ткань" охуеть блядь.
Аноним 16/10/20 Птн 07:06:31 51805891
>>518051
Чтобы была "чёрная дыра" нужно чтобы вокруг неё была обычная материя, если вселенная замкнута сама на себя, а не бесконечна, то непонятно что за её пределами, и как будет вести себя эта сжавшаяся хуйня при учёте того что вокруг неё непонятно что.
Это будет какой-то новый вид объектов, а не чёрная дыра.
Аноним 16/10/20 Птн 07:07:10 51805992
>>518058
>обычная материя
*обычное пространство
Аноним 16/10/20 Птн 07:51:52 51806193
>>518053
Ловко ты Эйнштейна обоссал, ничего не скажешь.
Аноним 16/10/20 Птн 08:34:21 51806394
>>518051
>Материю разметало в разные стороны? Разметало.
Ты это видимо неправильно представляешь. Скорее всего думаешь, Что материю разметало относительно какого-то "центра взрыва". Это не так.
Аноним 16/10/20 Птн 08:36:55 51806495
image.png 46Кб, 461x295
461x295
>>518046
>Раньше чем случится критическая деградация, человечество перейдёт к социализму, а затем и к коммунизму. А там уже половой отбор (в размножении) поправит интеллект популяции.
Когда у людей, которые подобный бред несут спрашиваешь "а какие предпосылки к этой хуйне"? Они начинают просто говорить "ну вот смотри, раньше был феодализм, потом стал капитализм"... мне хочется им вот эту картинку показать.
Аноним 16/10/20 Птн 11:59:22 51807296
>>518050
> то, что там было до инфляции
То что нельзя произносить вслух, а то вселенная сожмется
Аноним 16/10/20 Птн 12:05:55 51807397
giphy.gif 6728Кб, 480x360
480x360
Могли ли существовать феи и гномы размером с ложку и при этом быть одного с нами развития. Это вопрос про объем мозга и его возможности.
Аноним 16/10/20 Птн 13:09:25 51809198
Можно ли с помощью генной инженерии создать ДНК динозавра и его самого, ну и чтоб они могли сами размножаться?
Или это задачка на уровне Faster than the speed of light?
Аноним 16/10/20 Птн 13:11:55 51809299
>>518091
Как нехуй. Это гораааааздо проще, чем альбукерку надуть.
Аноним 16/10/20 Птн 13:58:13 518110100
>>518058
Какая нахуй вселенная, чёрная дыра, мань?
Ты в хруще живёшь, в Барнауле, а на кухне у тебя пакет для пакетов.
Аноним 16/10/20 Птн 16:04:22 518120101
>>518091
Можно создать мутанта внешне похожего на динозавра и показывать в зоопарке. Если просто воскресить динозавра то он скорее всего задохнется в непривычной атмосфере или от инфекций
Аноним 16/10/20 Птн 17:01:56 518133102
>>518061
Очнись, додзиси, ты хуй собачий, а не Эйнштейн.
16/10/20 Птн 18:47:33 518151103
image.png 1262Кб, 900x563
900x563
>>517798 (OP)
Во что мы превратимся во время сверхновой солнца? Предположить, что это случится завтра, мы превратимся в водородный пепел? Или распадёмся просто на составляющие независимые? 30% углерода, 60% водорода, 10% аскорбинки?
Аноним 16/10/20 Птн 20:36:19 518162104
>>518048
>Новая секта деградации мозга, забавно
Не достаточно оснований говорить о какой то абсолютной деградации. Однако если рассматривать человека как самоодомашненное животное, то действие стабилизирующего отбора на интеллект человека сократилось. С одной стороны это проводит к болезненным отклонениям и дисфункциям. С другой повышает изменчивость и даёт материал для эволюции.
Аноним 16/10/20 Птн 20:46:01 518165105
>>518162
>С другой повышает изменчивость и даёт материал для эволюции.
А смысл тогда лепетать о какой-то деградации? Про одомашнивание и легкость бытия это чушь, даже взять лиственного хиккана, он обрабатывает 300.000 гигабайт информации в наносекунду и шарит в кодинге, занимаясь флирансом. Деградация мозга невозможна в развивающимся обществе, это тебе не отвалившийся хвост и не бесполезные коренные зубы. Нужно корректно подбирать слова.
Аноним 16/10/20 Птн 20:53:33 518166106
>>518064
>Когда у людей, которые подобный бред несут спрашиваешь "а какие предпосылки к этой хуйне"? Они начинают просто говорить "ну вот смотри, раньше был феодализм, потом стал капитализм"... мне хочется им вот эту картинку показать.
На этой картинке нет примера качественных изменений, революционных.
Объясню тебе проще. У капитализма есть проблемы. В рамках капитализма их не решить ибо они исходят из самой его сути. А попытаться их решить без сохранения сути, то уже будет не капитализм. Проблемы капитализма в перспективе фатальны для существования человечества. Потому что всякое достижение прогресса, капитализм обращает в орудие конкурентной борьбы Ну и вообще капитализм пожирает любые даровые блага.
Почему говорят что коммунизм неизбежен для человечества? Потому что есть только 2 способа его избежать:
1) Уничтожение человечества.
2) Уничтожение человечности.
Разъясню второй пункт. Человечность - это продукт экологической ниши человека, как существа творчески преобразующего природу. Когда человек перестаёт быть ориентирован на преобразование мира - он теряет человечность. То что мы ебём друг другу мозги рекламой впаривая говно, вместо того что бы "делать ракеты" - это путь к потери человечности. Путь к тому что общество замкнётся само на себе и на ритуалах поддержания положения групп. То есть станет автоматоном без перспектив развития.

То как частная собственность мешает прогрессу, наиболее отчётливо видно в области патентного и авторского права. Довольно очевидно, что чем раньше начать преобразования, тем меньше человечество потеряет из за того что старые нормы и законы душат прогресс. И наоборот, чем дольше оттягивать изменения и продлевать закон Микки-Мауса, тем разрушительнее будут нарастающие проблемы и тем более взрывоопасным будут изменения когда их уже будет невозможно откладывать.
Аноним 16/10/20 Птн 21:12:14 518168107
>>518120
>Можно создать мутанта внешне похожего на динозавра и показывать в зоопарке. Если просто воскресить динозавра то он скорее всего задохнется в непривычной атмосфере или от инфекций
Чем казуар - не динозавр?

>>518073
>Могли ли существовать феи и гномы размером с ложку и при этом быть одного с нами развития. Это вопрос про объем мозга и его возможности.
А зачем им это? Добывать нектар из цветов - много ума не надо.

>>518165
>А смысл тогда лепетать о какой-то деградации? Про одомашнивание и легкость бытия это чушь, даже взять лиственного хиккана, он обрабатывает 300.000 гигабайт информации в наносекунду и шарит в кодинге, занимаясь флирансом. Деградация мозга невозможна в развивающимся обществе, это тебе не отвалившийся хвост и не бесполезные коренные зубы. Нужно корректно подбирать слова.
Ты наверно имеешь ввиду не все коренные зубы, а так называемые "зубы мудрости". Ты подобрал хорошую аналогию, но не развил её. Прежде чем их наличие станет редким отклонением, они будут нас мучить.Исчезнувшее давление стабилизирующего отбора позволяет зубам расти неправильно, приводя к дисфункции жевания. Конечно современная медицина это лечит, удалением неправильных зубов. Примени эту аналогию последовательно и на интеллект человека. Очень многие ментальные функции сейчас имеют функцию "крайних моляров". Хорошо когда они есть правильные - это удобно. Хорошо когда их нет - это тоже удобно. Плохо когда они исчезли на пол шишечки и вбок.

>взять лиственного хиккана, он обрабатывает 300.000 гигабайт информации
Человек независимо от своего желания обрабатывает информацию постоянно. Если человек будет 10 лет бродить по пустыни, он научится различать тонкие оттенки песка и будет знать повадки практически всей пустынной живности.

Однако про что я вообще пишу. Дестабилизирующий отбор в отношении некоторых ментальных функций ведёт к их неправильной работе. Это создаёт проблемы. А ещё, это ведёт к потери адаптивности. Идиот и умный человек в комфортном городе имеют практически равную адаптированность, по меньшей мере пока живут по проверенным сценариям. Но стоит случится чем нибудь их разрушающему, как дебилы начинают драться за туалетную бумагу! А случись что посерьёзнее, убьются о какую нибудь хуйню. А умственно здоровый человек приспособится к изменению уклада.
Аноним 16/10/20 Птн 21:18:16 518170108
>>518166
>На этой картинке нет примера качественных изменений, революционных.
Чел, индукция это не аргумент. Всё. Вся твоя графомания ненужна.

>Почему говорят что коммунизм неизбежен для человечества? Потому что есть только 2 способа его избежать:
>1) Уничтожение человечества.
>2) Уничтожение человечности.
Нет, коммунизм это и есть уничтожение человечности. Потому что эгоизм и жадность, это свойства человека, которые нельзя отнимать. Если их отнять, будет уже не человек нахуй.

>То как частная собственность мешает прогрессу, наиболее отчётливо видно в области патентного и авторского права.
Да, внезапно самый большой технический прогресс в странах с самой строгой патентной политикой. Охуенно "мешает". Калашников мог бы быть миллиардером, если бы родился не в совковой помойке.
Аноним 16/10/20 Птн 21:45:32 518171109
>>518168
Казуар это страус, даун.
>>518168
А зачем ты вообще что-то пишешь.
>Если человек будет 10 лет бродить по пустыни, он научится различать тонкие оттенки песка
Бедуины всю жизнь ходят по пустыне, что-то не слышал про оттенки песка. Где воду насосать через толще песка знают разве что.
> Идиот и умный человек в комфортном городе имеют практически равную адаптированность
Идиот лежит на кроватки в дурке и принимает еду с ложечки, пуская слюни, потому что у него тяжелая степень слабоумия. Я не думаю что это можно назвать адаптацией и сравнить с нормой.
>дебилы начинают драться за туалетную бумагу! А случись что посерьёзнее, убьются о какую нибудь хуйню
Случаи нападений и убийств среди умственно-отсталых, психически больных и нормальных людей примерно равны.

Аноним 16/10/20 Птн 21:46:56 518172110
>>518171
>А зачем им это? Добывать нектар из цветов - много ума не надо.
>зачем ты вообще что-то пишешь
Аноним 16/10/20 Птн 23:36:37 518183111
>>518170
>>Почему говорят что коммунизм неизбежен для человечества? Потому что есть только 2 способа его избежать:
>>1) Уничтожение человечества.
>>2) Уничтожение человечности.
>Нет, коммунизм это и есть уничтожение человечности. Потому что эгоизм и жадность, это свойства человека, которые нельзя отнимать. Если их отнять, будет уже не человек нахуй.
Противопоставление эгоизма и альтруизма верно для недоразвитой индивидуальности. А для развитой индиидуальности является ложным. То что обыватели понимают под словом эгоизм - является близоруким, недоразвитым, звериным эгоизмом.

>>То как частная собственность мешает прогрессу, наиболее отчётливо видно в области патентного и авторского права.
>Да, внезапно самый большой технический прогресс в странах с самой строгой патентной политикой.
Попробуй использовать в размышлениях категорию времени. Когда то патентное право было прогрессивным. Оно и сейчас способствует тому что технологии не засекречивают. Но оно мешает многие достижения применять на благо человечества.
>Охуенно "мешает". Калашников мог бы быть миллиардером, если бы родился не в совковой помойке.
Ты хочешь заявить что капитализм вознаграждает всех кто изобрёл что то полезное миллиардами денег? (можешь сам загуглить "умерщие в нищите изобретатели") Но вообще при чём здесь это? Миллиарды отсыпанные кому то в карман никак не относятся к техническому прогрессу. Не являются ни его фактором ни его мерилом.

Бостон. 29 марта. FINMARKET.RU - Право собственности на научную разработку - краеугольный камень инновационной экономики. Закон защищает его, чтобы корпорации могли вкладывать миллиарды долларов в исследования, а потом имели возможность вернуть их, реинвестировать прибыль в новые исследования и немного заработать на хлеб.

Так выглядит теория, но в реальности все совсем не так. Срок действия большинства патентов, выдаваемых в США, - 20 лет, и этот срок предопределяет выбор направления научных исследований, которые будут получать финансирование.

Фактически, корпорации выбирают не те направления, где может произойти настоящий прорыв, а те, где срок клинических испытаний будет меньше, а период монопольного владения рынком - больше.

То, что ими движет именно такая мотивация, доказали Хейди Уильямс из Массачусетского технологического университета, Эрик Будиш из Университета Чикаго и Бенджамин Роин из Юридической школы Гарварда которые исследовали, как патенты влияют на развитие инноваций. Для примера они взяли рынок разработок лекарств от рака. Уильямс до этого изучила рынок исследований генома человека.

Вывод в обеих работах один и тот же: защита интеллектуальных прав приводит к сокращению инноваций и мешает корпорациям финансировать прорывные фармакологические исследования.

https://www.interfax.ru/business/298489

Компания Celera владеет правом собственности на результаты расшифровки генома человека, она завершила свои исследования чуть раньше, чем был завершен международный некоммерческий проект "Геном человека". Предположим, что Pfizer изобрела уникальный тест на поиск какой-либо болезни, для тестирования которого нужно исследование генома и выявление соответствующих маркеров, ассоциированых с этим недугом.
В идеальном мире компании бы начали переговоры и договорились о совместных тестах. Но в реальном мире с высокими транзакционными издержками переговоры зайдут в тупик и инновации никогда не будет произведены.
Международный проект по исследованию генома человека был начат в 1990 году и закончен в 2003 году. Celera начала свое исследование в 1999 году и закончила в 2001 году.
С 2001 по 2003 годы Celera имела право на защиту генов, которые она открыла. Тогда компания могла продавать лицензии на их разработку.
Право собственности Celera привело к значительному сокращению исследований и разработок в этой сфере. К 2009 году на каждый ген, открытой Celera, приходилось 1,2 публикации, а на остальные - 2,1 публикации.
Лишь 3% генов Celera были использованы в различных диагностических тестах против 5,4% генов, не принадлежащих Celera.
Различные тесты показывают: тот факт, что Celera "владела правом на гены", сократило исследования и разработки на 20-30%.

Аноним 16/10/20 Птн 23:42:43 518186112
>>518171
>Казуар это страус, даун.
Насколько же у дурачков вроде тебя всё плохо с юмором.
>>>518168
>А зачем ты вообще что-то пишешь.
>>Если человек будет 10 лет бродить по пустыни, он научится различать тонкие оттенки песка
>Бедуины всю жизнь ходят по пустыне, что-то не слышал про оттенки песка. Где воду насосать через толще песка знают разве что.
А я слышал.
>> Идиот и умный человек в комфортном городе имеют практически равную адаптированность
>Идиот лежит на кроватки в дурке и принимает еду с ложечки, пуская слюни, потому что у него тяжелая степень слабоумия. Я не думаю что это можно назвать адаптацией и сравнить с нормой.
Идиот может спокойно работать контент-менеджером, системным администратором, установщиком окон водителем, ну или хотя бы охранником.
>>дебилы начинают драться за туалетную бумагу! А случись что посерьёзнее, убьются о какую нибудь хуйню
>Случаи нападений и убийств среди умственно-отсталых, психически больных и нормальных людей примерно равны.
Я не про тестовый абстрактный уровень интеллекта пишу. А про тот самый интеллект который даёт адаптивность, верную оценку действительности и творческий подход. То есть интеллект как взаимосвязанный и уравновешенный комплекс высших психических функций. А про эти ваши IQ и прочую скорость реакции.
Аноним 16/10/20 Птн 23:56:18 518190113
измерение 6083f[...].mp4 8059Кб, 1920x1080, 00:00:43
1920x1080
>>518172
>зачем ты вообще что-то пишешь
Потому что я могу.

Если ознакомится с происхождением человека. То в своей эволюции люди шли по многим путям, но никогда не шли до предела. Люди вышли в савану и приспособились, но даже павианы приспособились лучше. Биологический вид занявший специализированную нишу, обычно обречён. Полагаю что вымершие виды людей потому и вымерли, что угодили в ловушку специализации.

Возвращаясь к феечкам. У мелкого животного "нет задач" требующих сложного ума, ему достаточно инстинктов. то демонстрируют насекомые. Однако известно что существовал карликовый вид людей, с средней взрослой массой мозга 400 грамм. Эти люди умели добывать огонь и производить орудия. Мы не знаем пределов минитюаризации разумной нервной системы. Но знаем что некоторые очень мелкие мухи или клещи, не помню уже точно кто, сливает все клетки нервной системы в одну распределённую по всему телу. И заменяет межклеточную передачу сигналов внутриклеточной.

Технически разумные феи может и могли бы существовать, но у них должны быть большие предки. А к измельчанию должен быть эволюционный стимул, при этом не снижающий требований к умственным способностям.
Аноним 17/10/20 Суб 01:13:33 518199114
>>518186
>Насколько же у дурачков вроде тебя всё плохо с юмором.
Шутливо поссал тебе на ебало
>А я слышал.
А я нет
>Идиот может спокойно работать контент-менеджером, системным администратором, установщиком окон водителем, ну или хотя бы охранником.
Шутя серанул тебе на ебасос
>верную оценку действительности
Что ты лепечешь. Я как будто оказался на семинаре тянши и слушаю ахинею про духовный рост и саморазвитие.
> То есть интеллект как взаимосвязанный и уравновешенный комплекс высших психических функций. А про эти ваши IQ и прочую скорость реакции.
Ты однозначно бывал на этих семинарах. Тебе не в сцай, а куда нибудь в секту. Как минимум твою речи ненаучны.
Аноним 17/10/20 Суб 01:20:45 518200115
image.png 809Кб, 827x465
827x465
>>518190
> Люди вышли в савану и приспособились, но даже павианы приспособились лучше.
Что за хуйню ты несешь, приспособились отнимать еду у людей и жрать на помойках? Самый приспособленный человек если смотреть на уровень жизни и рост популяции.
>Возвращаясь к феечкам. У мелкого животного "нет задач" требующих сложного ума,
Ты в саване что ли рос и с культурой не соприкасался. Вон там на гифке фея разговаривает с человеком. Мы не про насекомых говорим
Аноним 17/10/20 Суб 01:57:33 518204116
>>518058
А с какой стати чему-то ещё сжиматься, кроме материи?
Частицы материи притягиваются друг к другу. Так из облаков газа звёзды и формируются звёзды.
Это вот я могу понять причину, по которой материя станет сжиматься. Взрывом её разметало, но по итогу, из-за взаимного притяжения частиц, всё опять сожмётся. В чёрную дыру.
Если же ты считаешь, что какая-нибудь там ПАРУСИНА вселенной, ВЕЛЬВЕТ космоса или там ФЛАНЕЛЬ времени начнёт тоже куда-то съёбываться, то тебе неплохо было бы пояснить: с какой такой стати.

>>518063
Да как заебало уже про этот центр взрыва. Носитесь с этой, вообще никак не относящейся к вопросу, максимой как с писаной торбой.
Пускай, пускай не было центра взрыва. Пускай. Окей. Пускай. Окей. Норм. Не было.
Но это не играет вообще никакой роли. Потому что чёрные дыры [оказавшиеся достаточно близко] сливаются в одну. И тут неважно, был ли центр у большого взрыва или не было. Совершенно неважно. Вообще. Никак. Ни капельки. Не писечки.

Если материю сильно сжимать, она превратиться в чёрную дыру. Чёрные же дыры [оказавшиеся достаточно близко] сливаются в одну. И по итогу, если материя сжимается, хоть в бесконечном количестве центров, должна получиться одна большая чёрная дыра. Будет ли она в центре вселенной или будет с краю - поебать. Никакого отношения к прозвучавшему вопросу это не имеет. Так что можно уже заткнуться по поводу того, что у вселенной нет центра.
Аноним 17/10/20 Суб 03:10:09 518206117
>>518204
>с какой такой стати.
"С стати" наблюдаемых явлений этого.
Аноним 17/10/20 Суб 06:10:57 518215118
Существует ли какая-то статистика об уровне интеллекта и доступности тех или иных областей в науках. Допустим меня заинтересовала физика, но у меня 100 iq, до какой степени я смогу понимать, а потом наступает порог где нужен вычислительный аппарат помощнее
Аноним 17/10/20 Суб 07:02:16 518216119
>>518204
>Но это не играет вообще никакой роли.
>чёрные дыры [оказавшиеся достаточно близко] сливаются в одну.
Если их две, да. Если три, тоже да... надо тебе говорить, что не будет у тебя черных дыр, если во всем пространстве равномерно будет увеличиваться плотность?
>Если материю сильно сжимать, она превратиться в чёрную дыру.
Упустил главную суть, надо иметь не просто сжатую материю, а большой градиент энергии/массы/плотности. Чтобы в одном месте нихуя, а рядом очень плотно.
Аноним 17/10/20 Суб 09:46:20 518222120
>>518215
Айсикью это как средняя температура по больнице, не очень качественный тест для оценки способностей человека в специфических областях. С физикой и математикой это особенно заметно. Даже если ты с 150 айсикью, то ты вполне не сможешь сдвинутся дальше школьного курса, а какой-нибудь аутист с 70 аьскью интуитивно понимает когомологии.
А есть еще химия, для большинства это что-то запредельное, хотя химиков трудно назвать сверхразумами.
Аноним 17/10/20 Суб 10:23:34 518231121
>>518222
Я тебя понимаю частично. Будучи пиздюком дома упарывался квантовой физикой, а в школе физика и химия казались мне грубыми и тупыми темами, с двойки на тройки перепрыгивал.
Аноним 17/10/20 Суб 11:52:05 518234122
изображение.png 59Кб, 321x320
321x320
>>517798 (OP)
Есть тут выпускники МАИ из макдональдса? Какой мудак придумал измерять скорость летательных аппаратов в махах, нахуя, что это дает и почему ему не насрали на голову?
Я понимаю еще, когда оцениваю скорость каких то потоков вокруг крыла или в сопле из носа. Тут ламинарное там турбулентное, хуе-мое. Одно дело, если бы это была какая-то инварианта, которая не менялась бы от высоты. А тут бля скорость 6М. Где наухй? В стратосфере? На уровне моря? Какой в этом смысл? Выебываться умными единицами измерения? А по ебалу? Ну, блядь, хорошо решили что высота 10км скорость звука триста - отсоси у тракториста, умножили. Во-первых нельзя было сразу без выебонов этих написать 1800м/с на высоте 10км? Сука, не так умно, блядь?
Аноним 17/10/20 Суб 12:25:01 518237123
>>517798 (OP)
Как фридайверы задерживают дыхание на 5 и более минут? Мозг же без длительного доступа кислорода начинает повреждаться, не говоря уже о том, что они не просто стоят на месте а плавают, то есть тратят энергию и сжигают еще больше кислорода
Аноним 17/10/20 Суб 13:34:11 518242124
>>518199
Установщик окон, спокойно.
Аноним 17/10/20 Суб 13:52:08 518244125
>>518242
>Установщик окон
Я бы мечтал о такой работе, 50к изи рубить, для ДС3 неплохо
Аноним 17/10/20 Суб 13:59:54 518245126
>>518234
А мы моряки по морю ходим и вообще в узлах считаем. Так что не выебывайся.
Аноним 17/10/20 Суб 14:03:46 518247127
image.png 322Кб, 500x375
500x375
>>518200
>> Люди вышли в савану и приспособились, но даже павианы приспособились лучше.
>Что за хуйню ты несешь, приспособились отнимать еду у людей и жрать на помойках?
Люди тоже начинали с того что группой отбирали добычу ухищников.
>Самый приспособленный человек если смотреть на уровень жизни и рост популяции.
Это сейчас.
>>Возвращаясь к феечкам. У мелкого животного "нет задач" требующих сложного ума,
>Ты в саване что ли рос и с культурой не соприкасался. Вон там на гифке фея разговаривает с человеком. Мы не про насекомых говорим
Я тебе привёл эти примеры, но кажись ты не понял того что я пытался тебе объяснить. Хотя я прямо это написал. Тот кто эволюционирует как небольшое животное, не имеет эволюционных стимулов к развитию разума, в силу мелкости и простоты манипуляционных задач. То есть что бы головной мозг начал усложнятся до качественного переустройства в разумный, нужно набрать некоторую массу, при которой при которой дальнейший количественный рост не даёт преимуществ, однако эволюционный стимул к тому что бы становится умнее присутствует. Например в силу сезонности различных источников пищи и необходимости групповых взаимодействий для её добычи.

>гифке фея разговаривает с человеком
Формат GIF не предусматривает звук. У приложенного файла формат mp4.

>>518215
>Существует ли какая-то статистика об уровне интеллекта и доступности тех или иных областей в науках. Допустим меня заинтересовала физика, но у меня 100 iq, до какой степени я смогу понимать, а потом наступает порог где нужен вычислительный аппарат помощнее
Текстовые IQ тесты определяют общую развитость и в какой то мере эрудированность. При том под развитостью понимается культурная адаптированность. Результаты графических IQ тестов коррелируют с ловкостью мышления.
Высокий IQ позволяет легче усваивать знания. А успешное освоение каких нибудь знаний вызывает гормональное подкрепление и тем самым улучшает усвоение новых знаний. Это как с умением читать. Для тех кто читает по слогам - чтение это трудный процесс, который должен оправдываться быстрым вознаграждением. Иначе баланс удовольствий станет отрицатльным. А те кто умеет читать быстро и не шевеля челюстями, читают очень легко и поэтому способны осиливать большие книги со сложной информацией, им для продолжения чтения не требуется постоянного подкрепления.

Но вообще, разговоры о голой ловкости ума имеют мало смысла. Имеет значение сила вооружённого ума. Вооружённого логикой и общими знаниями всех основных наук.

>>518234
>Одно дело, если бы это была какая-то инварианта, которая не менялась бы от высоты. А тут бля скорость 6М. Где наухй? В стратосфере? На уровне моря? Какой в этом смысл? Выебываться умными единицами измерения? А по ебалу?
Это так же как измерение расстояния в парсеках. Удобная для наглядного сравнения величина.

>>518237
>Как фридайверы задерживают дыхание на 5 и более минут? Мозг же без длительного доступа кислорода начинает повреждаться, не говоря уже о том, что они не просто стоят на месте а плавают, то есть тратят энергию и сжигают еще больше кислорода
При тренировках увеличивается объём лёгких, количество гемоглобина в крови и количество миоглобина в мышцах. А кроме того, я сам занимаясь гипнозом узнал что головной мозг может работать экономично, отключая многие высшие функции.

>>518199
>Я как будто оказался на семинаре тянши и слушаю ахинею про духовный рост и саморазвитие.
тебе противна сама идея всестороннего развития человека?
>>518199
>Ты однозначно бывал на этих семинарах. Тебе не в сцай, а куда нибудь в секту. Как минимум твою речи ненаучны.
Любой человек должен уметь менять пеленки, планировать вторжения, резать свиней, конструировать здания, управлять кораблями, писать сонеты, вести бухгалтерию, возводить стены, вправлять кости, облегчать смерть, исполнять приказы, отдавать приказы, сотрудничать, действовать самостоятельно, решать уравнения, анализировать новые проблемы, побросать навоз, программировать компьютеры, вкусно готовить, хорошо сражаться, достойно умирать.
Специализация — удел насекомых.

Роберт Хайнлайн, Достаточно времени для любви, 1973


Аноним 17/10/20 Суб 14:06:37 518248128
>>518244
>>Установщик окон
>Я бы мечтал о такой работе, 50к изи рубить, для ДС3 неплохо
А почему ты не мечтаешь о такой работе, в которой ты реализуешь творческие способности? Такой работе которая будет не истощать твои физические и духовные силы, а приумножать их?

Я так полагаю, что ты дурачок не с критичным, а с экономичным (энергоэффективным) мышлением, так как вместо него у тебя триггеры. Полагаю сейчас ты стриггерился на слово "духовные".
Аноним 17/10/20 Суб 14:08:29 518249129
>>518247
>Формат GIF не предусматривает звук. У приложенного файла формат mp4.
Даун, не позорься. Эта доска для элиты двача, а у тебя
> преобладает конкретно-описательный тип мышления, в то время как способность к абстрагированию почти отсутствует. Составляет трудность охватывание ситуации целиком, и обычно они улавливают только внешнюю сторону событий. Бедный словарный запас. Хотя может быть и достаточно большой словарный запас, но речь при этом всё равно бедна и однообразна
Аноним 17/10/20 Суб 14:09:18 518250130
16028043420610.png 324Кб, 811x676
811x676
>>518248
> дурачок не с критичным мышлением
Аноним 17/10/20 Суб 14:30:16 518256131
>>518248
Полагаю ты сейчас стригеррился с того что я сказал что полагаю ты как даун постоянно используешь слово полагаю, полагаю.
Аноним 17/10/20 Суб 15:37:31 518265132
Какие есть сейчас государственные программы или стартапы по разработке вечного двигателя?
Аноним 17/10/20 Суб 15:38:18 518266133
Аноним 17/10/20 Суб 17:18:10 518286134
>>518247
В случае с парсеком нет такой бредятины. Парсек равен вполне конкретному количеству метров.
Аноним 17/10/20 Суб 17:49:19 518288135
Возможно ли вывернуть плоскость наизнанку, как выворачивают сферу в мат.топологии?
Аноним 17/10/20 Суб 17:50:24 518289136
>>518248
Этот человек не лишён мозга.
мимо
Аноним 17/10/20 Суб 17:59:28 518291137
>>518165
>Деградация мозга невозможна в развивающимся обществе
Ещё как возможна. Уже сейчас горожанин в принципе не осознаёт, как ему обеспечить психический комфорт, у многих проблемы с телесным. Обработка сенсорных сигналов в полной жопе. Поизучай вопрос прежде, чем постить.
Западная культура прокачивает абстрактное мышление, но заставляет сасат всё остальное.

>>517958
>эволюционно закрепляется отвращение к инцесту на генетическом уровне.
Африканцам это расскажи.
>Про сексуальное влечение к родителям - это антинаучный бред Фрейда, ничем не подтвержденный.
Фрейд, по всей видимости, сталкивался с насилием отцов над детьми обоих полов, но не смог правильно это протрактовать. Нинужности психоанализа это не отменяет, да.
Аноним 17/10/20 Суб 18:00:59 518292138
>>518206
Наблюдаемых явлений чего?

>>518216
>если во всем пространстве равномерно будет увеличиваться плотность?
Почему во всём пространстве будет увеличиваться плотность? Да ещё и равномерно?
Аноним 17/10/20 Суб 18:42:52 518296139
>>518289
Гуманитарные мозги тут нам не нужны
Аноним 17/10/20 Суб 18:48:32 518299140
>>518291
> как ему обеспечить психический комфорт
Потреблять развлекательный контент, игры, таблетки, вещества, алкоголь, секс, рыбалка, тысячи способов.
>Обработка сенсорных сигналов в полной жопе
Получше чем у многих животных, включая кошек и собак. Как думаешь что слышит кошка когда играет симфония Моцарта? Просто шумы. Поизучай вопрос прежде, чем постить.

>Западная культура прокачивает абстрактное мышление, но заставляет сасат всё остальное
В случай с тобой соглашусь, ты туповатый и чсвшный.
>Фрейд, по всей видимости, сталкивался с насилием отцов над детьми обоих полов, но не смог правильно это протрактовать
Он скинул листву после того как написал свои труды. Типичный великовозрастный инцел с кучей комплексов и неврозом.

Аноним 17/10/20 Суб 19:11:44 518306141
>>518292
>Наблюдаемых явлений чего?
"Сжимания с какой-то стати чего-то кроме материи".
Аноним 17/10/20 Суб 19:25:54 518308142
Во время оргазма и секса в целом какие нейромедиаторы выделяются в большей степени? Дофамин и окситоцин? А эндорфины как?
Аноним 17/10/20 Суб 19:30:13 518309143
>>518292
>Почему во всём пространстве будет увеличиваться плотность? Да ещё и равномерно?
Я описал тебе то, что происходит. если отматывать время впять.
Аноним 17/10/20 Суб 19:51:05 518315144
>>518306
Например?

>>518309
С чего время-то отматываться вспять станет? С чего?
Я могу допустить, что частицы притягиваться станут друг к другу, потому что они и так уже притягиваются. Но вот время, время-то почему отматываться вспять станет? Из-за чего?
Аноним 17/10/20 Суб 20:22:12 518317145
Всем привет! Не нашел тут треда географов/геологов или кого-то подобного, так что спрашиваю тут. У меня поллиноз(аллергия на цветение), достаточно сильный. Реакция идет на деревья средней полосы России - березы, клены, дубы лещины(ореховые деревья), рябина. Вот в чем вопрос: я устал перебирать страны для переезда по-штучно, есть ли сервис для отбора территории по растениям, или просто могут ли мне местные геологи подсказать на какие страны ориентироваться, на какие широты смотреть?
Аноним 17/10/20 Суб 20:26:27 518318146
Аноним 17/10/20 Суб 20:37:25 518321147
>>518318
природа Магадана не подходит, березы там растут, хоть и карликовые
Аноним 17/10/20 Суб 20:58:11 518323148
>>518249
>Даун, не позорься. Эта доска для элиты двача, а у тебя
>преобладает конкретно-описательный тип мышления, в то время как способность к абстрагированию почти отсутствует.
Одно из основных ментальных отличий человека от обезьяны - это гораздо большая занудность.

>>518256
>Полагаю ты сейчас стригеррился с того что я сказал что полагаю ты как даун постоянно используешь слово полагаю, полагаю.
Ну я телепат, могу телепать и могу не телепать. Откуда мне точно знать что ты думаешь?

>>518286
>В случае с парсеком нет такой бредятины. Парсек равен вполне конкретному количеству метров.
Всякое уточнение метра будет изменять парсек. Хотя конечно его можно и отвязать конкретного расстояния, но тогда он станет маня-единицей.
Когда то преодоление скорости звука было значимым достижением. Поэтому величина и закрепилась.

>>518289
>Этот человек не лишён мозга.
>мимо
Я про то пишу, что мамкины фалософы кукарекают про "эгоистичную сущность человека" по которой надо жить. Но подлинный человеческий эгоизм они в себе не развили. Их эгоизм звериный, детский, недоразвитый, близорукий. Их эго не качественное, не способное выйти за пределы собственной тушки, как в пространстве так и во времени (конкретное время жизни).
Их эгоизм ничтожен по сравнению с эгоизмом тех кто хочет всем причинять добро.

>>518291
>>эволюционно закрепляется отвращение к инцесту на генетическом уровне.
>Африканцам это расскажи.
Это всего лишь культурная традиция. Люди увидели что от далёких браков рождается больше здоровых людей и стали это использовать.

Аноним 17/10/20 Суб 21:02:13 518324149
>>518323
>Этот человек не лишён мозга
Шиз, прекращай сам с собой
Аноним 17/10/20 Суб 21:49:38 518332150
>>518288
Плоскость ты имеешь ввиду чисто R 2 односвязное открытое многообразие?
Да, возможно, но для этого надо выходить в 4-мерное.
Аноним 17/10/20 Суб 21:55:33 518333151
>>518315
Искривление движения солнечных лучей возле тяжёлых объектов, изменение течения времени и т.д.
Аноним 17/10/20 Суб 22:05:04 518334152
>>518333
То есть из того, что солнечные лучи искривляются возле тяжёлых объектов, сторонники гипотезы большого отскока делают вывод, что вселенная начнётся ещё раз?
Аноним 17/10/20 Суб 22:11:41 518338153
>>518334
Шта? Ты же спрашивал скакого хуя считают что сжиматься могут не только частицы "типа реальные".
Аноним 17/10/20 Суб 22:57:03 518363154
>>518338
Нет. У тебя видно особый сорт выборочной слепоты, при котором видишь только тот вопрос, на который способен ответить.

Я спрашиваю: С какой стати [при гипотетическом сценарии большого отскока] станет сжиматься что-то ещё кроме материи?
На что ты мне отвечаешь: "С стати" наблюдаемых явлений этого.

Ну ты понел, да?

Если ты только сейчас подключился, то уточняю, в этой ветке обсуждается конкретно гипотеза большого отскока.
Аноним 18/10/20 Вск 00:29:18 518370155
>>518363
>станет сжиматься что-то ещё кроме материи?
Так уже сжимается, ...

А, ты про это

Да, кстати интересно, то про что я писал это только расширение, т.е. растягивание,
оно получается из-за гравитации
но
расшипение вселенной которое идёт, это тоже расширение, но тут почему-то говорят что тут уже нужна антигравитация
Типа тёмная энергия это сорт оф антигравитация, или по крайней мере проявляет её свойства.
Непонятно.
Аноним 18/10/20 Вск 03:19:03 518380156
>>518370
*типа и от гравитации расширение/растягивание пространства, и от антигравитации расширение
А сжатие тогда вообще отчего будет?
непонятно
Аноним 18/10/20 Вск 03:28:43 518381157
Аноним 18/10/20 Вск 04:00:50 518384158
>>518370
Не, не сжимается. Наоборот, вселенная разлетается во все стороны. Красное смещение от окружающих галактик однозначно об этом говорит.
А тёмная энергия это то, из-за чего скорость разлёта не то что не замедляется, а ещё и ускоряется.

>>518380
Как я и объяснял в первом посте, в теории всё должно начать сжиматься из-за гравитации. Типа как камень, который подбросили вверх, но рано или поздно он всё равно ёбнется вниз, потому что притягивается к земле. И мол нынешнее состояние вселенной, это вот пока камень ещё не ёбнулся.
Просто частицы обладающие массой притягиваются друг к другу. То есть если в космическом пространстве будут висеть атомы какого-нибудь газа и на них не будут воздействовать никакие иные силы, то они потиху-потиху станут притягиваться друг к другу. Так и формируются звёзды.
Однако в нашей вселенной слишком мало вещества, оно слишком сильно разобщено, его слишком разметало взрывом, чтобы оно начало обратно стягиваться, силы взаимного притяжения слишком слабы в таких масштабах. Ещё и тёмная энергия подливает урана в реактор.

В целом гипотеза большого отскока больше притягательна, нежели убедительна.
Есть в ней что-то подспудно религиозное. Люди как будто хотят видеть в ней какую-то надежду. Непонятно правда на что, но видно нашим обезьяньи мозгам больше нравится сценарий вечного повторения, нежели какой-нибудь там тепловой смерти вселенной, по итогу которого жизнь не сможет существовать ни в какой форме.

Но реальности поебать на на наши надежды. Кажется читал у Вайнберга, что на каждую ядерную частицу приходится миллиард фотонов.

Выходит наша вселенная соткана из света. А значит смерть неизбежна.
Аноним 18/10/20 Вск 10:17:21 518397159
>>518384
>Красное смещение от окружающих галактик однозначно об этом говорит.
Гойлактики стоят на месте, пространство пухнет и возникает иллюзия разбегания
Аноним 18/10/20 Вск 17:58:28 518469160
>>517798 (OP)
>Тред тупых вопросов
Энергия - это система отчёта?
Аноним 18/10/20 Вск 17:58:51 518470161
Аноним 18/10/20 Вск 20:09:49 518489162
>>518397
Да нет же, это голактеки разбегаются и из-за этого создаётся иллюзия распухания пространства.
Аноним 18/10/20 Вск 20:48:36 518490163
Эскобар.mp4 574Кб, 480x360, 00:00:13
480x360
Аноним 19/10/20 Пнд 17:24:33 518583164
Какие самые вредные ежедневно потребляемые продукты отрицательно влияют на наш мозг?
Аноним 19/10/20 Пнд 17:33:59 518586165
>>518583
Все, что продается по акции в пятерочке.
Аноним 19/10/20 Пнд 17:38:33 518587166
>>518583
Кислород, он вызывает окислительное разрушение мембран клеток и межклеточного скелета.
Аноним 19/10/20 Пнд 17:43:09 518588167
Аноним 19/10/20 Пнд 17:49:15 518591168
А вот есть допустим оксида урана.

Ядро урано распалось(если одну молекулу рассматривать), что с оксидом урана стало?
Аноним 19/10/20 Пнд 18:03:55 518595169
>>518583
Никотин, алкоголь, газ выхлопных труб.
Аноним 19/10/20 Пнд 18:04:50 518596170
>>518586
Если не чистить кожуру с лимонов и прочих фруктов, которые покрывают парафином, то вредно
Аноним 19/10/20 Пнд 18:07:06 518597171
>>518591
Оксид урана формуют кристаллическую решетку. В случае его распада возникнет дефект в решетки.
Если попытаемся взять одну одинокую молекулы тут зависит от канал распада. В случае урана спонтанный распад превращает уран в торий. Вот и получим оксид тория. Большую часть энергии уносить альфа частица, так что молекула не распадется.
Аноним 19/10/20 Пнд 19:07:56 518601172
115763.jpg 125Кб, 1210x957
1210x957
Испорченные куриные яйца в воде всплывают, а свежие тонут. Как я понимаю, в испорченных вырабатыватся сероводород. Но он вырабатывается из желтка и белка, яйцо замкнуто скорлупой, т.е. общая масса всего вещества не меняется. Так почему тогда всплывает?
Аноним 19/10/20 Пнд 19:30:31 518602173
>>518601
> яйцо замкнуто скорлупой
не герметично
Аноним 19/10/20 Пнд 20:38:59 518607174
2020-10-19203540.jpg 37Кб, 673x199
673x199
>>517798 (OP)
А что значит "Спонтанное деление" в описании направлений деления урана?
И почему о нём так мало и непонятно написано, т.е. вообще никакой инфы.
Типа вообще на любые осколки может распасться?
Аноним 19/10/20 Пнд 21:13:58 518617175
>>518607
> Типа вообще на любые осколки может распасться?
Не совсем. Там одни элементы возникают чаще, у других шансов появиться меньше.
Аноним 19/10/20 Пнд 21:23:04 518621176
>>517800
Деньги не создаются из воздуха, там весь смысл в том что их ограниченное количество.

И почему все начали хотеть ей владеть(желание владеть и даёт ценность, цену любым вещам), что и дало ей ценность-дело в безопасности.
Т.е. любой маняцарёк, будь то трамп или пыняпидарас, может просто взять и начать печатать себе деньги страны которой правит, обворовывая тем самым друг держателей этих денег.
Аноним 19/10/20 Пнд 21:58:32 518625177
что такое бозон Хигса,что все с ним носятся,в чем прикол?
Аноним 19/10/20 Пнд 21:59:43 518626178
Какой метод сегодня больше даёт профита науке - теоретический или эмпирический?
Аноним 19/10/20 Пнд 23:24:42 518640179
Тупой вопрос: хули на оп-пике Ступа?
Аноним 19/10/20 Пнд 23:37:03 518643180
>>518625
> что такое бозон Хигса
Фейк.
> что все с ним носятся
Ну типа вау круто, что-то там открыли, в коллайдере что-то ебануло, стрелочка что-то показала, натянули сову на теорию, бабло отмыли. Что-то между фото черной дыры и лохнесским чудовищем.
Аноним 19/10/20 Пнд 23:39:39 518644181
Почему градиент указывает на именно на максимум функции, а не на минимум или вообще не идет куда-то в бок?
Аноним 19/10/20 Пнд 23:41:17 518645182
>>518640
Очки ботаника добавляют интеллекта, типа ученый
Аноним 19/10/20 Пнд 23:44:14 518646183
image.png 755Кб, 800x518
800x518
image.png 64Кб, 290x270
290x270
>>518626
Теория дает околонулевой профит без эмпирического подтверждения.
Вот типичный теоретик
Аноним 19/10/20 Пнд 23:54:55 518648184
>>518621
> пыняпидарас, может просто взять и начать печатать себе деньги страны которой правит
Ты ошибаешься, хохол, так нельзя
Аноним 20/10/20 Втр 00:32:02 518650185
>>518648
Да что ты говоришь, америка себе печатает, пыня печатает, лукашенко печатает, все печатают.

Это только холопам нельзя, а баринам можно.

А ты чего порвалась пидарашка? Я кстати из беларуси.
Аноним 20/10/20 Втр 00:37:49 518651186
>>518595
>Никотин
Ну, это вот спорно. Никотин активизирует ацетилхолиновые рецепторы. Что положительно влияет на память и внимание, препятствует развитию альцгеймера.
Если бы не канцерогенность, табак бы прописывали всем и каждому, в качестве лёгкого ноотропа и для профилактики нейродегенеративных заболеваний.
>алкоголь
А вот это да, просто пиздец. Даже если абстрагироваться от всех остальных чудовищных побочек, вроде кровоизлияний в мозг, и взять только один тот факт, что он хуёво влияет на память - этого уже достаточно, чтобы никогда к нему не притрагиваться. Потому что память и способность обучаться это чуть ли не одно и то же.
Аноним 20/10/20 Втр 00:43:42 518652187
>>518646
Без внятной теории нет даже примерного представления, какие эксперименты ставить и для чего.

Перефразируя Айера, можно сказать, что теория без эмпирики - пуста, а эмпирика без теории - слепа.
Аноним 20/10/20 Втр 01:04:44 518656188
>>518651
Вместе с никотином от сигарет много чего плохого поступает. Курение помогает решать простые задачи, но мешает решать сложные.
Аноним 20/10/20 Втр 01:06:50 518657189
>>518652
Хотя план эксперимента и может быть, но не обязательно, сделан в рамках какой-либо теории.
Аноним 20/10/20 Втр 01:29:37 518662190
>>518656
Не понял о чём ты. Воздействие никотина заставляет тебя мыслить на процентик, на писечку яснее. Совершенно неясно как это мешает решать сложные задачи.
Аноним 20/10/20 Втр 01:35:28 518663191
>>518657
Любому эксперименту предшествует теоретическое осмысление. Иначе он просто ничего не будет значить.
Эксперимент должен что-то доказать, опровергнуть или продемонстрировать. И вот это вот то, что он должен доказать, опровергнуть или продемонстрировать и есть результат теоретического осмысления.
Предположение и опровержение. Без предположения экспериментатору просто неизвестно что делать. И самое главное зачем.
Аноним 20/10/20 Втр 01:40:51 518664192
>>518662
В общем длительное курение вызывает утоньшение коры головного мозга. Может даже никотин тут не причем, а это побочное явление от убитых легких, смолы и тд.
Аноним 20/10/20 Втр 02:30:22 518666193
Note Bene.png 2489Кб, 1280x1024
1280x1024
Тупому треду - тупой вопрос
Чтобы с планеты было видно её спутник как на пикрилейтед, какие должны быть характеристики заявленных тел?
Аноним 20/10/20 Втр 03:01:32 518672194
>>518666
Ну типа как луна наша если, то поближе. Или побольше. Желательно двойную планету где шары близки по размеру
Аноним 20/10/20 Втр 03:02:30 518673195
Аноним 20/10/20 Втр 03:11:13 518676196
>>518666
Вообще я туповат и под пивас тебе печатаю, но что-то мне подсказывает, что вид как на пикрелейтед может возникнуть только в случае, если на скриншоте на небе мы наблюдаем не спутник, а планету, а сами наоборот находимся на ее спутнике.


Ты залезаешь в sci, но ты делаешь это без уважения, ты пытаешься найти смысл в фэнтези...
Аноним 20/10/20 Втр 03:16:57 518678197
>>518666
Ну тут скорее наоборот это ты должен на спутнике жить. Например одном из спутников Юпитера.
Аноним 20/10/20 Втр 10:50:58 518696198
>>518602
А если бы герметично? И литр воды весит столько же, сколько в сумме кислород и водород, нужные для его получения (отбросив примеси в воде и проч.)
Аноним 20/10/20 Втр 10:53:07 518697199
>>518650
>Я кстати из беларуси.
Блядорус, мой жопу и в койку, завтра на митинх.
Аноним 20/10/20 Втр 11:41:47 518701200
>>518696
Большинство подобных вопросов которые тут задают гуглятся в мейл ответах. Не понимаю зачем их сюда высирают если даже обсуждение не несет никакой интриги. Если герметичное то будет тонуть. А вот еслиб тухлое герметичное яйцо грелось от внутреннего бурления и там образовалась микроцивилизация, которая сильно фонила гамма-излучением, то наверно яйцо просело бы по массе через столетия.
Аноним 20/10/20 Втр 11:56:40 518705201
Аноним 20/10/20 Втр 12:00:28 518707202
image.png 1614Кб, 1594x900
1594x900
>>518705
Просто воспользовался силой
Аноним 20/10/20 Втр 12:29:17 518714203
>>518646
Почему тогда все ублажились от ОТО изначально, если это была чистая теория? Ведь колебания пространства нашли гораздо позже. >>518663
А каким образом происходит именно прорыв? Ну то есть момент, когда теория себя полностью себя исчерпала? Делается теоретическая гипотеза, которая должна быть проверена опытом?
Аноним 20/10/20 Втр 13:52:23 518759204
Кто здесь знает о теории плазменных диспозиций Воргафта-Жизнера?
Аноним 20/10/20 Втр 13:56:21 518761205
>>518714
ОТО не работает, го создадим теорию о черной материи.
Аноним 20/10/20 Втр 14:31:25 518764206
>>518761
Почему не работает? Кто проверял?
Аноним 20/10/20 Втр 14:34:04 518765207
>>518761
Гойлактики странным образом движутся и не рассыпаются. Я когда пиздюком был еще до черных материй догадывался что-то тут не так, не может такая хуйня просто взять и образовать галактику из облака пыли.
Аноним 20/10/20 Втр 14:36:53 518766208
Аноним 20/10/20 Втр 14:54:26 518769209
У меня практический вопрос - увеличится ли влажность в 15м комнате, и на сколько, если поставить таз с водой на пол? Или два? В комнате очень сухо. Увлажнитель хочу брать хороший, а денег пока нет, вот и думаю чем пока можно обойтись.
Аноним 20/10/20 Втр 15:03:22 518771210
>>518769
намочи простынки и развесь, можно вентилятор подрубить
Аноним 20/10/20 Втр 15:06:24 518772211
>>518626
Больше всего знаний получается из рассчетов.
Аноним 20/10/20 Втр 15:09:57 518773212
>>518237
Обычный человек не может задержать надолго дыхание из-за слабой диафрагмы. У ныряльщиков сильная и выносливая диафрагма. Где-то видел оценки, что человек может провести в сознании 30 минут на одном вдохе.
Аноним 20/10/20 Втр 15:43:38 518777213
ДНК - двойная спираль с нуклеотидами, ген - сплошные участки нуклеотидов в ней, которые отвечают за синтез белка или РНА. Сама "нить" ДНК очень длинная - 2-3 метра. Она "упаковывается" в ядре эукариотических клеток (нитка наматывается на гистоны). Результат этой упаковки - хромосомы. Вопрос: почему пар хромосом 23? Эта нить ДНК режется перед упаковкой?
Аноним 20/10/20 Втр 16:06:29 518779214
>>518765
Ну тут тогда вообще вся научная парадигма тогда не работает, если я правильно понял. Поэтому пока и впихнули этот костыль под названием "тёмная материя", поставив себе найти этот ебучий ВИМП.
Аноним 20/10/20 Втр 16:08:08 518780215
>>518771
Двачую, поставь таз с водой, рядом с ним вентилятор. Повесь на вентилятор ткань, чтобы она опускалась в воду и включай.
Аноним 20/10/20 Втр 16:10:06 518781216
>>518780
Можно ещё в этот тазик сесть. Если пойдут пузыри - значит человека лишили анальной девственности, проще говоря «проткнули».
Аноним 20/10/20 Втр 16:13:40 518782217
>>518781
Человек ещё может пукать.
Аноним 20/10/20 Втр 16:23:47 518783218
>>518780
А если жиденько посрать на вентилятор, то образуется особая ламповая атмосфера
Аноним 20/10/20 Втр 16:29:59 518785219
Аноним 20/10/20 Втр 17:17:46 518788220
16016514582501.mp4 2349Кб, 960x540, 00:00:40
960x540
Аноним 20/10/20 Втр 17:57:37 518797221
Аноним 20/10/20 Втр 18:03:33 518801222
>>518777
Короче. Причуды эволюции.
ДНК на самом деле не линейная нить. Помимо генов в хромосомах есть некодирующие белки последовательности, которые важны на этапе деление клетки и экспрессии генов.
Самые заметные это теломеры и центромеры - специальные участки, за которые могут цепляется специальные белки, которые растаскивают хромосомы при деление или определяют начало/конец хромосом.
В добавок есть "защищенные " участки хромосом. С них производиться трансляция специальных РНК, которые идут на строительство рибосом, а так же специальных заглушек РНК, связывающие с белками или с самой ДНК. Последние РНК уже регулируют экспрессию генов, собственно стыкуют пары хромосом, а так же помогают чинить поломанные ДНК. Эти участки так же обеспечивает один из механизмов репродуктивной изоляции.
Эти защищенные участки для каждой хромосомы в процессе эволюции вырабатывались отдельно. В итоге каждая пара хромосома генетически самодостаточная. Все процессы идут в пределах одной хромосомы. Никто не собирает в одну общую нить. Впрочем есть процессы когда одна хромосома срастается с другой или лишняя появляется, но это это отельный разговор.
Аноним 20/10/20 Втр 18:57:10 518807223
>>518714
Когда проёбывается объяснительная сила теории. Когда теория становится неспособна объяснить наблюдаемые явления.
Ну вот как например возник квантмех:
Всё в физике было ясно, всё было известно, всё изучено. Единственно что не могли понять, а вот отчего железячка, когда её греешь, начинает светиться красным, если ещё сильнее греешь, то белым, а если совсем пиздец раскочегарил, то аж синим.
И ни одна из существующих теорий не могла объяснить это явление. Так Планк и понял, что имеющиеся в наличии теории - не оч и попытался найти своё собственное объяснение. И ему это удалось. Так и возник квантмех.
Прорыв в науке происходит тогда, когда наука заходит в тупик.

Наука это такое зазеркалье. Что доктрине, учению или религии - смерть, то науку только разовьёт. Загонишь её в тупик, она прорвётся ещё в сто раз дальше. Разобьёшь в пух и прах её основания, на их руинах тут же вырастет ещё более устойчивая всеобъемлющая конструкция. Да она этому только и рада. Наука всячески приветствует критику, по существу на ней и строится. Предположение и опровержение. И от охуенного опровержения она выигрывает ничуть не меньше, чем от охуенного предположения.
А всё от того, что однажды отказалась от поиска абсолютов. И поэтому теперь её путь лежит в бесконечность.

Ах да, первая часть сообщения адресована не мне, однако тем не менее, может я ошибаюсь, но мне почему-то кажется, что ты не до конца понимаешь значение слова "теория". Теория это что-то, что связывает и объясняет множество фактов. То есть "чистой теории", т.е. теории не основывающейся ни на каких фактах, быть не может. Скорее всего то, что ты подразумеваешь, когда говоришь "чистая теория", это ты имеешь ввиду "гипотеза".
Аноним 20/10/20 Втр 19:21:18 518808224
>>518807
А какие там сейчас тупики есть?
Аноним 20/10/20 Втр 19:45:29 518809225
>>518807
>то доктрине, учению или религии - смерть, то науку только разовьёт. Разобьёшь в пух и прах её основания, на их руинах тут же вырастет ещё более устойчивая всеобъемлющая конструкция. Да она этому только и рада.
Ты сейчас буквально страничку пропагандонской методички пересказал, "нас ебут, а мы крепчаем". Полезные санкции. Никогда ещё ни доктрина, ни религия не умирали от такой хуйни, ну и наука, соответственно, тоже, потому что она сорт доктрины.
>>518808
Да вот кризис проверяемости и сам факт того, что наука религия. Фанатики бегают кругами и орут "ряяя", никакого только выиграли не намечается в перспективах.
Аноним 20/10/20 Втр 19:53:36 518811226
>>518809
>Да вот кризис проверяемости и сам факт того, что наука религия.
Бля, хуесос, съеби, не тебя спрашивали.
Аноним 20/10/20 Втр 20:21:56 518815227
>>518811
Вот я и говорю, порватки бегают и орут "замолчите, не хочу слышать, всё вы врёти".
Аноним 20/10/20 Втр 20:23:30 518817228
>>518815
Не рвись, верунская шлюха.
Аноним 20/10/20 Втр 20:24:47 518818229
>>518817
+15 индекса цитирования, говно.
Аноним 20/10/20 Втр 20:25:55 518819230
тер.png 20Кб, 501x284
501x284
>>518808
Выделил красным. Последняя деталь, последний кусочек пазла. Когда он будет найден, будет построена теория всего.
Аноним 20/10/20 Втр 20:27:59 518820231
>>518819
Ты сначала построй великое объединение.
Аноним 20/10/20 Втр 20:30:18 518821232
>>518819
>Последняя деталь, последний кусочек пазла.
Почему фотон волна и частица, напомни мне, я забыл.
Аноним 20/10/20 Втр 20:32:52 518822233
>>518821
А раньше ты знал?
Аноним 20/10/20 Втр 20:36:11 518823234
>>518822
Да, в школе рассказали, что так велел Б-г, как и гравитацию. Но вдруг у тебя научный ответ есть.
Аноним 20/10/20 Втр 20:45:14 518825235
>>518823
Да не, всё верно. Молодчинка. Скушай облатку.
Аноним 21/10/20 Срд 01:35:23 518842236
>>518809
>Фанатики бегают кругами и орут "ряяя", никакого только выиграли не намечается в перспективах.
У тебя слишком короткая жизнь или кругозор. Как муравей не может судить о форме предмета, по которому он ползёт. Ты думаешь наука за 10 лет делается? Ага разбежался. Изначально прорыв в физике привел к прорыву в инженерии. На этом построили новые приборы, и сделали следующий прорыв. Он же в свою очередь, сначала развил химию, которая придумала материалы, которые уже сделали возможным многие новые приборы и только потом опять физика поехала вверх. На следующем этапе, цепочка стала еще длиннее...
физика-инженерия-химия-компьютеры-физика... Сейчас из-за вычислительной мощности, мы можем позволить себе высматривать сигналы в шуме из петабайтов информации. Следующая цепочка видимо потребует рывок в еще одной науке, это может много что оказаться, это может быть прорыв в квантовых вычислениях, это может быть биологический прорыв (сейчас биология открыла для себя дивный новый мир точечного редактирования генов и возможности компьютерной обработки сотен тысяч геномов), ну и может будет какой-нибудь рывок в нейросетях (хотя сложно представить как они помогут, биологи хоть гениев выращивать смогут).
Аноним 21/10/20 Срд 03:59:30 518850237
>>517881
Как причина связана со следствием? Ну, сам подумай...

Любой процесс, да даже не процесс, а просто любой объект,
они СУЩЕСТВУЮТ не только в пространстве, но ВО ВРЕМЕНИ,
то есть, как-бы перетекают, в одну сторону, по оси четвёртого измерения - по оси времени,
из одного отрезка планковского времени: https://ru.wikipedia.org/wiki/Планковское_время
в другой отрезок планковского времени.

Предыдущий отрезок планковского времени, относительно текущего - является причиной состояния объекта (или процесса),
в текущий отрезок планковского времени.
В свою очередь, состояние объекта (процесса), на текущем отрезке планковского времени,
является следствием наличия причины, то есть состояния объекта (процесса) - в предыдущий отрезок планковского времени.
Или в два отрезка, или в три, или в стопицот отрезков.

Преобразование же причины в следствие (преобразование состояния объекта, или процесса) - протяжено во времени,
и имеет определённую закономерность.
Ну, а взаимосвязь между причиной и следствием, и есть эта закономерность, сама её структура,
это как структура программы, которая исполняется объектом (процессом) - просто по мере течения времени,
в процессе существования объекта (процесса). И разумеется, сама эта взаимосвязь тоже, протяжена во времени.

Если мысленно остановить время - остановится и исполнение программы,
следствие не будет достигнуто с момента запуска процесса причиной,
мысленно обрати время вспять, получишь преобразование следствия - в причину:
https://ru.wikipedia.org/wiki/Ретропричинность

Обрати время вообще, в планковскую эпоху https://ru.wikipedia.org/wiki/Планковская_эпоха
- получишь беспричинное, истинно-спонтанное её появление.
Если конечно, там всё не конформно, и не состоит из Эонов, как в теории Пенроуза. Ведь оно может быть и так: >>518213 →
Аноним 21/10/20 Срд 04:08:15 518851238
>>518850
Прежде чем пиздеть, докажи, что время существует.
Аноним 21/10/20 Срд 04:55:16 518854239
>>518851
Планковское время ежжи.
Аноним 21/10/20 Срд 12:48:48 518881240
>>518854
Которое просто частота между двумя событиями??
Аноним 21/10/20 Срд 15:03:38 518888241
>>518881
Короче это расстояние между событиями, которое коммутирует с гамильтонианом, величина которого на масштабе этого участка насколько большая, что уже должны сказываться эффекты ОТО. Но такой физики еще не завезли. Короче на этих интервалах заканчивается применимость нашей физики.
Аноним 21/10/20 Срд 15:27:32 518890242
>>518888
Я о том что времени как такового от этой фразы "планковское время" не появляется.
Аноним 21/10/20 Срд 16:11:12 518897243
>>518890
Время как и пространство не появляются из чего-то. Они самодостаточные сущности. Время это расстояние между событиями, которые мы можем как либо фиксировать. Физика задает закономерности между событиями и расстоянии между ними. Планковское время это минимальное расстояние между событиям, которые мы можем зафиксировать и различить(в пределах нашей физики).
Аноним 21/10/20 Срд 17:41:07 518911244
Сап, двач. Тут один вопрос.
Нужна конструкция буквально ПРОСТЕЙШЕГО ядерного реактивного двигателя. Мысль такова: несколько кусков не-очень-долгоживущего изотопа. Куски такого объема и размера, чтобы самим по себе им хватало охлаждения окружающим воздухом, т.е. чтобы реакция распада в таких объемах шла слабенько. Площадь поверхности большая, сиречь буквально понасверлено сквозных отверстий от верха до низа. В момент запуска эти куски сближаются хорошо и необратимо, это всё начинает пиздец как греться, и сквозь всю эту ебанину летит рабочее тело, не давая ей перегреться. Когда РТ в баке кончается, вся схема радостно испаряется.
1. В чем я обосрался?
2. Будет ли оно работать? Возможно ли из кучки более-менее жизнеспособных кусков изотопа с дырками при соединении получить условный один, который будет выделять дохуя тепла? Близкий к началу совсем уж пиздецовой цепной реакции, то бишь
3. Какие изотопы для всего этого подойдут?
Аноним 21/10/20 Срд 17:55:35 518912245
>>518469
>Энергия - это система отсчёта?
да или нет?
сюда что не заглядывают представители ВЦ, рассматривавших эту трактовку?
Аноним 21/10/20 Срд 18:00:33 518914246
>>518897
Все еще не появляется.
Аноним 21/10/20 Срд 18:02:27 518915247
>>518911
> В чем я обосрался?
в самом начале противоречие:
> ядерного двигателя
и
> не-очень-долгоживущего изотопа


>>>518911
>Когда РТ в баке кончается
Химическое топливо даст больше и дольше тягу, чем простой ядерный, так как есть множество сложностей (но реализуемых уже на существующем уровне техники коллективами специалистов) на пути получения ядерного лучше химического.
Аноним 21/10/20 Срд 18:02:47 518916248
>>518911
>вся схема радостно испаряется.
Ну конечно
И ты вместе с ним, а потом на территории под местом испарения идёт радиоактивный дождик
Аноним 21/10/20 Срд 18:07:10 518920249
>>518819
Да нее, есть куча всякой мелкой хуйню, которую объяснить не могут.
Не такой существенной даже как это то что ты выделил.
Аноним 21/10/20 Срд 18:10:23 518921250
>>518915
> в самом начале противоречие:
> > ядерного двигателя
> и
> > не-очень-долгоживущего изотопа
Чому? Суть ярд - рабочее тело идёт сквозь реактор, снимает с него тепло и уходит в сопло. Не-очень-долгоживущий - в плане, как плутоний, несколько (десятков) лет.
> Химическое топливо даст больше и дольше тягу, чем простой ядерный
А как запилить такой дрыгатель, чтобы имел адекватную тягу?
Алсо предполагается, что в качестве рабочего тела - вода, но это необязательно

Кстати, а можно в качестве рабочего тела использовать какой-нибудь тритий в жидком состоянии, чтобы он, проходя сквозь активную зону, ловил дохуя нейтронов и грелся дополнительно от собственного распада? Оно же так работает, лол? Я темой не занимался, не ссы, лучше побей.
Аноним 21/10/20 Срд 18:11:24 518922251
>>518916
Это в данном случае неважно, схема для применения в писанине, а не ирл.
Аноним 21/10/20 Срд 18:16:41 518925252
>>518922
Что это значит?

Понимаешь, у тебя пока описание уровня школьника-дауна котлрому мозг марвел и другие говнофильмы современные разжижил.

Типа от установки ничего не требуется по описанию, но при этом чтобы она работала, что значит "работала"-не уточняется.

Опиши нормально, если хочешь получить ответ, сейчас отвечать-не на что.

Можно просто написать "да" разве что.
Аноним 21/10/20 Срд 18:19:40 518927253
>>518922
Перечитал твой бред ещё раз

В принципе да, всё это так и работает просто.

Один вопрос
Зачем дырки?
Аноним 21/10/20 Срд 18:25:43 518928254
>>518925
Ты какой-то тупенький или толстый, что более вероятно, но тогда тебе в /б/.

Вопрос состоит в том, можно ли на основе подобия "пушечной" схемы атомной боньбы запилить реактивный двигатель, в котором рабочее тело будет эффективно охлаждать непосредственно кусок изотопа, немного не дотягивающий до критической массы. А задача пушечной схемы - просто удерживать куски урана/плутония/любого другого говна от активной реакции до, собственно, запуска двигателя.
Без полноценного реактора и прочей хуйни.
Аноним 21/10/20 Срд 18:27:55 518929255
>>518927
> Один вопрос
> Зачем дырки?
Чтобы повысить площадь поверхности этого куска говна, и чтобы он отдавал тепло рабочему телу более-менее равномерно по всему объему, не начав испаряться где-нибудь внутри с закономерным результатом.
Аноним 21/10/20 Срд 19:23:02 518936256
>>518928
>Без полноценного реактора и прочей хуйни.
Скорость реакции, внезапно, определяется в первую очередь окружающим веществом, а не только болванками изотопа. Это очень сложная задача, а у тебя даже общего представления нет работы реактивного двигателя и ядерных реакций.
Кстати в википедии смотрел Ядерный_ракетный_двигатель ?
Аноним 21/10/20 Срд 19:37:44 518937257
>>518936
> Кстати в википедии смотрел Ядерный_ракетный_двигатель ?
Естественно.
> а у тебя даже общего представления нет работы реактивного двигателя
Есть.
> и ядерных реакций.
Вот это только в общих чертах. Иначе бы я не спрашивал здесь в надежде найти более разбирающихся в этой теме.
Аноним 21/10/20 Срд 19:40:56 518938258
И вообще, это тред тупых вопросов или где? Сюда, по-моему, и приходят за ответами на вопросы по неизученным темам
Аноним 21/10/20 Срд 19:49:48 518942259
>>518937
>> а у тебя даже общего представления нет работы реактивного двигателя
>Есть.
Нету. Даже в химическом водород+кислород выделяется так много энергии, что приходятся подавать излишний водород для понижения температуры. Так что ничем не нагреть воду сильнее, чем есть в хим. двигателе. Повышение скорости истечения возможно лишь на водороде или гелии.
С другой стороны для максимальной скорости истечения должна быть специальная форма камеры сгорания/нагрева и сопла.

>> и ядерных реакций.
>Вот это только в общих чертах.
А тут без глубоких знаний можно лишь сказать, что в малом объёме можно получить огромную мощность для достаточного нагрева чего угодно, но размеры и скорость подачи газа должны быть точно выдержаны, иначе реактор перегреется или наоборот не даст нужную мощность.
Аноним 21/10/20 Срд 19:56:13 518944260
>>518938
>неизученным
Где тут неизученная тема?

И это тред не "посчтитайте за меня".
Аноним 21/10/20 Срд 20:03:00 518946261
>>518942
> Даже в химическом водород+кислород выделяется так много энергии, что приходятся подавать излишний водород для понижения температуры.
Я знаю. Только не излишний, лол; если память не изменяет, он потом ровно так же идет в камеру сгорания.
> Так что ничем не нагреть воду сильнее, чем есть в хим. двигателе.
Ладно, окей. Понял.
> Повышение скорости истечения возможно лишь на водороде или гелии.
Ладно. А более плотных альтернатив водороду и нет, видимо; жаль.
> но размеры и скорость подачи газа должны быть точно выдержаны, иначе реактор перегреется или наоборот не даст нужную мощность.
Окей. Значит, хуита, и какие-то системы контроля реакции все равно нужны. Понял.
Аноним 21/10/20 Срд 22:00:00 518982262
>>518914
Схулей самодостаточная сущность должна появляться, если она уже есть? Она появилась в планковскую эпоху.
Аноним 21/10/20 Срд 22:21:27 518992263
Интерференция волн в двухщелевом эксперименте проявляется только при определённом расстоянии между щелями и их размерами?
Аноним 22/10/20 Чтв 01:14:05 519019264
>>518992
нет, всегда, просто у картин видность плохая.
щели должны быть маленькими, чтоб можно было пользоваться приближением точечных вторичных источников, а расстояние между щелями мало, чтоб некогерентность излучения не мешала.
Аноним 22/10/20 Чтв 02:20:07 519027265
>>518992
Интерференция появляются, когда у нас есть когерентный источник.
Размер и расстояние щелей уже будут определять "размытость" картины интерференции.
Впрочем на больший щелях система становиться слишком чувствительна к возмущениям, нарушающая когерентность. Собственно поэтому опыт довольно сложно сделать кустарно на коленке.
Аноним 22/10/20 Чтв 02:51:41 519029266
>>519027
>>519019
Получается у частиц нет чёткой границы и они уходят своими "краями" в бесконечность по какому-то закону?
Пусть очень слабо после какого-то расстояния от "центра" но уходят?

Мне не нравится идея описания частиц как дуализм волна-частица
Аноним 22/10/20 Чтв 02:51:46 519030267
>>518982
Ты сказал? Докажи, что что-то там появилось.
Аноним 22/10/20 Чтв 03:04:39 519031268
>>519029
>Получается у частиц нет чёткой границы и они уходят своими "краями" в бесконечность по какому-то закону?
Типа того. Там немножко не так, но в принципе да, в бесконечность. И да, дуализма там нет. Там просто волна.
Аноним 22/10/20 Чтв 03:12:57 519032269
>>519031
>Там просто волна.
Я думал так написать, но потом подумал зачем уточнять "волна локализованная в пространстве, а не как обычная распространяющаяся во все стороны", типа зачем употреблять этот термин если он уже используется для другого.
А потом подумал что частицы не обязательно куда-то движутся в отличии от волны, могут и на месте стоять, и решил вообще не писать про волну.
Возмущение поля, впуклость или выпуклость трёхмерная, область напряжённости, но не волна

Так эта "расплывчатость размеров", если такую модель составить, она полностью повторяет щелевой эксперимент?
Аноним 22/10/20 Чтв 03:22:38 519033270
>>519032
>Возмущение поля, впуклость или выпуклость трёхмерная, область напряжённости, но не волна
Даже когда на месте стоит, там фаза волновой функции туды сюды "крутится" как волчок.
>Так эта "расплывчатость размеров", если такую модель составить, она полностью повторяет щелевой эксперимент?
Смотря как ты определишь "размер". Просто само понятие в его классическом понимании не применить уже к частицам.
Аноним 22/10/20 Чтв 05:06:33 519036271
>>517798 (OP)
Что мешает после тепловой смерти вселенной разряженному веществу стягиваться вместе под действием гравитации? Эффект казимира же недопустит идеальной размазанности вещества.
Аноним 22/10/20 Чтв 05:12:49 519037272
>>519036
Тепловая смерть это про другое.
Аноним 22/10/20 Чтв 05:55:43 519042273
>>519037
Ну немного перед ней.
Аноним 22/10/20 Чтв 07:52:20 519052274
>>519036
То что для тепловой смерти нужно сохранение энергии, а во вселенной энергия не сохраняется.
Аноним 22/10/20 Чтв 09:09:19 519064275
>>519030
>Ты сказал? Докажи, что что-то там появилось.
Ну, бля, если до планковской эпохи не было планковской эпохи, предшествующей ей,
то значит время появилось в планковскую эпоху - это очевидно же.

>>518851
>Прежде чем пиздеть, докажи, что время существует.
>>518854
>Планковское время ежжи.
>>518890
>времени как такового от этой фразы "планковское время" не появляется
А давай, я попробую доказать тебе, что время существует - "от обратного"?
Представь себе, что времени нет, есть просто пространство, трехмерное, и объекты внутри него.
Вселенная, неподвижна, короче. Вопрос? А вот СУЩЕСТВУЕТ ли такая Вселенная?
Нет, блядь, не существует. Потому что процесс сущестования, он... Ну... Как-бэ... Во времени направлен. Ололо.
Если времени нет, то нихуя нет. Следовательно, если что-то есть, и что-то существует, значит и время есть.
Поебнятена доказана, ежжи.
Аноним 22/10/20 Чтв 09:24:01 519070276
>>519064
>нет не существует
Не знаю о какой дичи вы говорите, но это мне режет глаза.
Время это просто мера изменения объектов, вот и все. Существовать и меняться это разные вещи.
Аноним 22/10/20 Чтв 10:34:14 519085277
16015587493490s.jpg 5Кб, 220x220
220x220
>>519070
> Существовать и меняться это разные вещи
Я меняюсь, следовательно я существую
Аноним 22/10/20 Чтв 10:51:42 519087278
Аноним 22/10/20 Чтв 11:11:49 519088279
>>519070
Процесс существования, направлен во времени, и если нет времени, то нельзя сказать что что-то существует.
Тот же, вышеописанный трехмерный объект, если и существует, то хотя-бы на одном отрезке планковского ВРЕМЕНИ. В-Р-Е-М-Е-Н-И. А если времени нет, то нихуя и не существует, кроме абстракции в голове у этого >>519087 .
Аноним 22/10/20 Чтв 11:15:56 519089280
>>519088
Кстати да, изначально, этот >>519087 здесь >>518851
вообще просил ДОКАЗАТЬ, что ВРЕМЯ, блядь СУЩЕСТВУЕТ.
Как ВРЕМЯ может СУЩЕСТВОВАТЬ, без ВРЕМЕНИ, блядь?
Если само понятие СУЩЕСТВОВАНИЕ, подразумевает наличие самого ВРЕМЕНИ.
Оно, это время, само в себе и существует, и это очевидно, ебать.
И эта очивидность, она как-бы ежжи доказательство.
Аноним 22/10/20 Чтв 11:16:14 519090281
>>519088
>процес с н аправлен
Оговорочка: НАШ процесс направлен

>то н ельзя сказать
НАМ нельзя напрямую НАБЛЮДАТЬ имея НЫНЕШНИЕ знания, говорить проецируя возможности, можно, как это и сделал

Я загуглил твоего планка, освежевал свою память, то о чем ты говоришь относится же к волнам всяким, свойствам НАШЕЙ с тобой реальности.


Аноним 22/10/20 Чтв 11:23:21 519091282
image.png 392Кб, 489x522
489x522
Аноним 22/10/20 Чтв 12:37:20 519095283
>>519089
>Если само понятие СУЩЕСТВОВАНИЕ, подразумевает наличие самого ВРЕМЕНИ.
На чём основано утверждение?
Аноним 22/10/20 Чтв 12:39:48 519097284
>>519052
На чём основано утверждение?
Аноним 22/10/20 Чтв 12:40:41 519098285
>>519036
> Эффект казимира же
А он тут причём?
Аноним 22/10/20 Чтв 13:56:40 519104286
>>519097
Ускоряющееся расширение
Аноним 22/10/20 Чтв 14:11:32 519107287
Аноним 22/10/20 Чтв 14:34:27 519108288
Аноним 22/10/20 Чтв 17:22:16 519119289
Анон, есть случайная величина u, которая равномерно распределена от 0 до N.
Если есть единственное наблюдение u =4, можно ли что-то сказать о N?
22/10/20 Чтв 17:45:01 519121290
>>519119
Только то, что N не меньше u.
Домашку делают в другом разделе.
Аноним 22/10/20 Чтв 18:30:56 519123291
>>519121
Спасибо.
это не совсем домашка
Аноним 22/10/20 Чтв 19:50:39 519129292
Аноны, я не знаю куда писать, так что спрошу тут: Есть ли какие-то хорошие разоблачения Рыбникова? Того самого, который ноль целковый и т.д. Просто вдруг стало интересно. На ютубе есть парочку видосов, но там уровень дискурса просто пиздец: ты не ученый, ты говно -- нет ты говно, а я великий математик. Хочется если честно чтобы даже такой дурачек как я выкупил все.
Аноним 22/10/20 Чтв 20:18:00 519132293
>>519129
>какие-то хорошие разоблачения Рыбникова
Его теория имеет предсказательную силу?
Аноним 22/10/20 Чтв 20:34:46 519137294
Квантовая теория постулирует случайные события?
Или отчего там Эйнштейн бугуртил?
Аноним 22/10/20 Чтв 20:38:44 519139295
Аноним 22/10/20 Чтв 21:07:00 519146296
>>519129
Как только слышишь чушь про всяких великих русов, которых подлые западожидомасонорептилоидопиндосы угнетают лживой наукой - сразу говоришь себе: "ебать он шизофрик" и идёшь по своим делам. Хули там разбирать?
Аноним 22/10/20 Чтв 21:13:10 519149297
>>519119
По теореме Байеса можно оценить вероятность различных исходных распределений.
Аноним 22/10/20 Чтв 21:14:55 519150298
>>519137
Да, бугуртил от рандома. Даже придумал парадокс, который бы ложил на лопатки кванты с помощью ограниченности света. Но кванты положили на лопатки СТО, сказав вилку в глаз или в жопу раз выбирай теперь что убрать локальность или реализм.
Аноним 22/10/20 Чтв 21:20:21 519151299
>>519146
Вдруг в его бреде есть крупица разума
Аноним 22/10/20 Чтв 21:38:19 519156300
>>519139
Рыбников Юрий Степанович
ВУЗ: МГАКХиС (ВЗИСИ), 1977, Факультет: Технологический, Кафедра: Химической технологии силикатных материалов
Изследователь, создатель Периодической таблицы электроатомов РУСов, Единой теории поля, методики построения электроструктур электроатомов, соединившей физику, химию, электричество, счёт РУСов (математику) в единую систему Знаний.
Изобрёл, разработал и внедрил в СССР порошковую полимерную покрасочную технологию.
Патентовед.
Преподавал в Московском государственном техническом университете Радиотехники Электроники и Автоматики (МГТУ МИРЭА), доцент.
Разоблачитель ложных установок и программ зомбирования в системе образования.
Аноним 22/10/20 Чтв 21:47:56 519157301
>>519150
>бугуртил от рандома
Так а что конретно рандом доказывает?
Аноним 22/10/20 Чтв 22:07:38 519158302
Мистическая связь между близнецами как объясняется с точки зрения науки?
Аноним 22/10/20 Чтв 22:09:49 519159303
>>519158
Пиздежем беспруфным или совпадением.
Аноним 22/10/20 Чтв 22:12:16 519160304
>>519159
Для совпадений слишком часто у одной пары близнецов происходит проявление связи. Для пиздежа слишком много пар рассказывают про это.
Аноним 22/10/20 Чтв 22:15:02 519161305
>>519160
>происходит проявление связи
Безпруфное.

>рассказывают
Нахуй. Слова это пустая хуета.

Это как психиатрия и психология.
Аноним 22/10/20 Чтв 22:15:43 519162306
>>519158
>как объясняется с точки зрения науки?
Ко-ко-ко, кудах, кукареку. Как обычно объясняется, в общем.
Аноним 22/10/20 Чтв 22:16:22 519164307
>>519160
>Для пиздежа слишком много пар рассказывают про это.
В гороскопы веришь?
Аноним 22/10/20 Чтв 22:19:31 519165308
>>519164
Нет, не верю. Своим двоюродным братьям верю - зачем им пиздеть (вместе с кучей людей в интернете), не ебу.
И вообще, чем стул со "связанными" близнецами хуже стула мирового заговора близнецов по наёбу населения?
Аноним 22/10/20 Чтв 22:23:17 519166309
>>519165
Зачем ты одну маняфантазию пытаешься доказать другой маняфантазией?
Аноним 22/10/20 Чтв 22:23:18 519167310
>>519165
>Своим двоюродным братьям верю - зачем им пиздеть (вместе с кучей людей в интернете), не ебу.
Когнитивные искажения.
>стул со "связанными" близнецами хуже стула мирового заговора близнецов по наёбу населения?
В смысле? Наука однозначна по этому вопросу.
Аноним 22/10/20 Чтв 22:31:38 519168311
>>519167
>Когнитивные искажения.
В общем, наука не знает, потому врёти. Ок, не страшно, не впервой.
>В смысле?
>>519166
>Зачем ты одну маняфантазию пытаешься доказать другой маняфантазией?
Если связи нет, но зачем-то тысячи близнецов пиздят про одно и то же, про то, чего нет? Значит, заговор. Представь завтра все астрономы будут про нло говорить. Это или факт прилёта нло, или заговор астрономов.
Аноним 22/10/20 Чтв 22:34:20 519169312
image.png 207Кб, 309x376
309x376
>>519168
>Мистическая связь
Наука этим не занимается. Замени мистику на другое слово и попробуй еще раз. А так тебе в магач
Аноним 22/10/20 Чтв 22:35:42 519170313
>>519169
Ну сам и замени. Можешь даже убрать его. Это эпитет, а не научный термин.
Аноним 22/10/20 Чтв 22:44:51 519171314
>>519168
Эксперименты то есть. Они однозначно говорят. Связи нет всё хуйня. Близнецы просто любят загоняться и подмечать совпадения.
Аноним 22/10/20 Чтв 22:51:20 519172315
>>519170
Приведи конкретные примеры
Аноним 22/10/20 Чтв 22:54:15 519173316
>>519168
>но зачем-то тысячи близнецов пиздят про одно и то же, про то, чего нет?
Потому что пиздоболы и вниманиебляди.
>>519168
>Представь завтра все астрономы
А ты им-прифы! И они тебе пруфы со своих астрономических приборов.
Иначе идут нахуй.
Астрономы на то и астрономы, потому что пиздят с пруфами.
Аноним 22/10/20 Чтв 23:01:46 519174317
image.png 1410Кб, 800x800
800x800
>>519173
Сразу люди в черном по всему миру посетят обсерватории и астрологи начнут исчезать загадочным образом или забывать про свои наблюдения.
Аноним 23/10/20 Птн 00:00:54 519177318
>>519137
Вообще нет.
В квантовой теории состояние системы образуют линейное пространство, а всякие величины, которые в классике мы считали состояние(координаты/время/импульс/энергия/даже мать его действие) становятся операциями с этим пространством. И нельзя взаимооднозначно связать пространство с операциями над ним. В итоге мы не можем восстановить истинную картину объективной реальности из имеющегося. То что мы имеем по сути субъективную информацию. Вселенная проводит шершавым по нашим мечтам об объективной реальности и ньютоновском детерминизме. Вот с этого бугурдил Эйнштейн и многие кто.
Короче все уходит в бесполезный философский срач о интерпретациях. Вот в мейнстиримной интерпретации вселенная прикрывается рандомом. В других нет.
Аноним 23/10/20 Птн 00:31:24 519179319
>>519177
>Вселенная проводит шершавым по нашим мечтам об объективной реальности и ньютоновском детерминизме.
Но как то что по каким-то там эмпирическим формулкам считающим средние значения правильно следует что "вселенная проводит детерменистам шершавым"?
Хотели посчитать просто средние значения, не рассматривая каждое конретное взаимодействие-взяли и посчитали.

Это ж вообще никак не опровергает детерменизм, и не показывает что именно так вселенная и работает.
Аноним 23/10/20 Птн 01:00:14 519183320
>>519179
Там фишка с парадоксом ЭПР. Который заставляет тебя либо от реализма отказываться либо от локальности.
Аноним 23/10/20 Птн 01:34:53 519184321
>>519183
Но как невозможность измерить ввиду того что измерением вносишь погрешность-доказывает то что детерменизма нету?

Ну не можешь ты измерить, твои проблемы, и что с того? Детерменизм как был, так и остался.(если он был)
Аноним 23/10/20 Птн 01:45:04 519187322
>>519183
Ну вот почитал я описание парадокса в википедии.

Суть проблемы как я понял не в том что опыт провели, и импульсы не подчинились закону сохранения импульса, а в том, что типа сверзсветовая мнгновенная связь якобы между частицами?
Но бля схуяли вообще такой вывод?
Они жу до этого распались, и уже полетели каждая в своём направлении и со своим импульсом.

Такое ощущение что квантовая механика пытается опровергнуть детерменизм так: "детерменизм невозможен, потому что он невозможен".


А, и зачем вообще квантовой механике для существования нужно выполнение неопределённости гейзенберга?
Это же не какая фундаментальная херня что ты не можешь измерить что-то маленькое не повлияв. Ну не можешь и не можешь, от того что ты это не измерил оно не перестало существовать или происходиь, вообще нихера тне ищмеряй и ничего не изменится(ну кроме того что ты внесёшь изменение измерением, лол)
Аноним 23/10/20 Птн 02:44:38 519188323
Мы где?
Аноним 23/10/20 Птн 02:48:26 519189324
Аноним 23/10/20 Птн 02:55:39 519190325
>>519095
На самом значении самого слова "существование":
https://ru.wiktionary.org/wiki/существование
В частности, на самом первом значении:
>Действие по значению гл. существовать; пребывание в мире
Слово пребывание, подразумевает то, что что-либо имеет место быть, в течении некоего времени.
Или вот ещё: https://ru.wiktionary.org/wiki/существовать
>находиться в процессе жизни
Очевидно, что любой процесс, направлен во времени.
>присутствовать
Очевидно, что в течении некоего времени.

А также, и на том, что обратное - исключено.
Как можно вообще представить себе, существование чего-либо , без существования времени?
Хотя-бы один отрезок планковского ВРЕМЕНИ, он должен-то быть, чтобы что-либо существовало в течении него.

Или нет? Если нет, то докажи обратное!
А если ты просто считаешь что нет, то хотя-бы - приведи хоть какие-то примеры.
Аноним 23/10/20 Птн 03:07:19 519191326
>>519090
>Оговорочка: НАШ процесс направлен
Тащемта любой процесс во времени направлен.
Ведь это процесс, блядь, он же протекет за счет ресурса - в результат, в течением времени, ёпт.

>НАМ нельзя напрямую НАБЛЮДАТЬ имея НЫНЕШНИЕ знания,
>говорить проецируя возможности, можно, как это и сделал
Что сказать-то хотел?

>Я загуглил твоего планка, освежевал свою память, то о чем ты говоришь относится же к волнам всяким, свойствам НАШЕЙ с тобой реальности.
Ты не планка гугли, ты величины планковские, лучше загугли, в частности - планковское время. И глянь там, по верхам.

>освежевал
>>519091
Лолблять. Анус себе освежуй, волнистый.
Аноним 23/10/20 Птн 07:28:08 519199327
>>519184
>невозможность измерить ввиду того что измерением вносишь погрешность-доказывает то что детерменизма нету?
Парадокс ЭПР это про то, что дело не в "измерением вносишь прогрешность", а про то, что важен сам факт измерения, даже если ты косвенное измерение делаешь (не взаимодействуя с самим объектом).
>Такое ощущение что квантовая механика пытается опровергнуть детерменизм так: "детерменизм невозможен, потому что он невозможен".
С детерминизмом у КМ проблемы нет. Проблема в том, что откуда одна частица знает, как повела себя вторая. Тут либо нужно принять, что они обменялись инфой быстрее света (а это досвиданья локальность), либо что есть какой-то внутренний механизм, который решает, какой результат измерения ты получишь (это опровергается как раз экспериментально), а значит третья возможность такая, частицы до измерения действительно не имели конкретного значения, а "решили это" во время измерения, при этом "магичиским" образом скоррелировали свои результаты.

Самый стандартный пример, ты отправляешь в разные стороны спутанные электроны. Они летят в разные стороны, ты хочешь измерить спин у первого. Делаешь несколько тысяч одинаковых опытов для верности, получаешь набор "верх-вниз-верх-верх..." короче полный рандом, спрашиваешь у коллеги на другой части земного шара, который второй измерял одновременно с твоим, он тебе выдает ровно противоположные результаты. Ага... получается, электроны либо заведомо "договорились" во время разлета (что опровергается экспериментами), либо решили на месте, но каким-то образом скорректировали свои результаты. Вот тут и возникает вилка возможности, мы уверены, что пока он к тебе летел, у него не было конкретного значения спина (можно даже сделать аналог двухщелевого эксперимента, где щель будет выбираться в зависимости от направления спина, у тебя появится интерференция, электрон залетал в обе щели одновременно), получается электрон определился со спином в момент измерения, хорошо, возможно мы своим влиянием на него так подействовали. Тогда возникает вопрос, почему измеряя один электрон, второй получает конкретный спин, на второй то мы не действовали никак, и надо либо признать, что они обменялись инфой, либо например признать, что в реальности реализовались обе возможности, и корреляция произошла на уровне мира.
В момент измерения спина, "мир" расщепился на две альтернативы, в одной ты увидел спин вверх, в другой ты увидел спин вниз. Аналогично мир расщепился и для твоего коллеги, но так как вы в одном мире вроде живете, то коллега увидевший спин вверх, не мог бы оказаться с тобой, который тоже увидел его вверх. Но это уже рассказ об интерпретациях, они не влияют на предсказательную силу КМ.

На уровне КМ это можно описать как |коллега>|ты> (|↑↓>+|↓↑>) состояние системы до измерения, ты, коллега и электроны не взаимодействуйте, поэтому я спокойно умножаю ваши векторы состояния. Система электронов описывается состоянием (|↑↓>+|↓↑>) которое суперпозиция двух вариантов. После измерения, ты провзаимодействовал с электронами, поэтому придется раскрыть скобки для тебя.
|коллега>(|ты увидевший вверх>|↑↓>+|ты увидевший вниз>|↓↑>) а дальше придется сделать аналогично для коллеги, как видишь, получится суперпозиция состояний,
|коллега увидевший вниз>|ты увидевший вверх>|↑↓> + |коллега увидевший вверх>|ты увидевший вниз>|↓↑>
Как видишь, варианта, где ты и коллега, наблюдали бы одинаковый результат нету.
23/10/20 Птн 09:17:21 519204328
>>519199
>спрашиваешь у коллеги на другой части земного шара
Посредством классического канала связи. На этом можно и заканчивать.
Аноним 23/10/20 Птн 10:32:04 519207329
Сможет ли человечество в текущем виде избавиться от проблем со спиной эволюционным способом? Отбора же сейчас уже считай никакого не ведётся. Нам теперь так и придётся страдать до скончания веков из-за не до конца адаптированного к прямохождению позвоночника?
Аноним 23/10/20 Птн 10:55:10 519210330
Кажется, я не умею думать самостоятельно.
Читаю учебник -- всё понимаю. Стоит начать делать что-то самому - сразу начинаю перебирать все варианты не думая, даже не думаю толком, нет того мыслительного процесса, который происходит во время чтения книги.
Это как-нибудь фиксится?
Аноним 23/10/20 Птн 10:57:08 519211331
>>519210
Для уточнения контекста: я имею ввиду программирование.
Каждую задачу можно решить 100500 способами и я в итоге метаюсь туда-сюда, думаю, как это можно сделать, даже не думаю толком.
Аноним 23/10/20 Птн 10:59:42 519212332
Аноним 23/10/20 Птн 11:23:56 519215333
Что по вашему эффективнее для развития человечества: скрещивать тупых с умными, чтобы выравнивать население или скращивать умных с умными, а тупых с тупыми, чтобы не портить умные гены, а делать их лишь лучше?
Аноним 23/10/20 Птн 11:43:27 519220334
>>519215
Сначала докажи что интеллект определяется генами.
Возможно быдлогопник при рождении был умнее тебя, просто до 2 лет как маугли не развивался. Смотрел в потолок из люльки и всё.
А потом следовал дурному примеру, и бухал, мозг атрофировался.
Аноним 23/10/20 Птн 11:45:53 519221335
>>519207
Если считать эволюционным способом генную инженирию то мб.
Аноним 23/10/20 Птн 11:46:54 519222336
>>519211
Ну значит ты хуевый программист раз сам не можешь думать, ищи что-нить другое.
Аноним 23/10/20 Птн 12:14:36 519226337
>>519220
> Сначала докажи что интеллект определяется генами.
Негры
>Возможно быдлогопник при рождении был умнее тебя, просто до 2 лет как маугли не развивался.
В своей среде он может быть вполне смышленым и пользоваться авторитетом.
Аноним 23/10/20 Птн 12:16:13 519227338
Аноним 23/10/20 Птн 12:22:30 519229339
>>519220
>Сначала докажи что интеллект определяется генами.
Двачую, врождённость может и оказывает влияние, но так сильно отличаются условия формирования, включая детство, что на их фоне генетическая предрасположенность незаметна.
Аноним 23/10/20 Птн 12:24:57 519230340
>>519227
Капча фильтрует <70 iq
Аноним 23/10/20 Птн 12:34:32 519231341
>>519222
Лол, по-твоему, все с рождения должны всё уметь?
Аноним 23/10/20 Птн 13:14:20 519239342
>>519231
Все предопределено. Раз ты уже задаешь такие тупые вопросы это уже означает что ты не вывозишь. Когда человек на своем месте он не задает никаких вопросов.
Предрасположенность либо есть либо нет.
Аноним 23/10/20 Птн 13:19:45 519242343
>>519215
Избавляться от тупых путём убирания их финансирования за деньги не тупых.
Т.е. выпиливанием нахуй социализма.
Аноним 23/10/20 Птн 13:21:06 519243344
35a4f110-89bd-4[...].jpg 752Кб, 2032x3528
2032x3528
>>519242
Тогда тупые начнут убивать умных
Аноним 23/10/20 Птн 13:29:16 519246345
>>519199
>Парадокс ЭПР это про то, что дело не в "измерением вносишь прогрешность", а про то, что важен сам факт измерения, даже если ты косвенное измерение делаешь (не взаимодействуя с самим объектом).
Как это доказывается? чем важен? Частицам не надо связываться чтобы сказать друг-другу свой импульс и направление, они у них уже были после разлёта

>Проблема в том, что откуда одна частица знает, как повела себя вторая.
Ебать шиза, зачем частице это знать?

>электроны либо заведомо "договорились" во время разлета (что опровергается экспериментами

Как опровергается? Что за эксперимент?

Зачем их "спутанными" тогда назвали, если они никак друг с другом не взаимодействовали?


Аноним 23/10/20 Птн 13:32:38 519247346
>>519190
>Очевидно, что в течении некоего времени.
Что очевидно? Потому что ты написал "очевидно"? Очевидно нет.

Какого времени? Что за время? Почему время выражено как частота двух событий? Если время фундаментально, почему оно не выражено через фундаментальные константы?

Раз время выражено через события, то эти события быэудут происходить и без выражения из них времени.

СОБЫТИЯ первичны, они происходят, а потом лысые обезьяны уже приписывают к ним какое-то там придуманное ими время.
Аноним 23/10/20 Птн 13:33:30 519248347
>>519239
Если это и называется "предрасположенностью", то это абсолютная предрасположенность. Ты какой-то максималист мамкин.
Аноним 23/10/20 Птн 13:33:51 519249348
>>519243
Схуяли ты такую маняфантазию высрал?

Если тупых не защищать то их быстро захуярят.
Аноним 23/10/20 Птн 13:34:54 519250349
>>519249
На картинку нажми, олигофрен малолетний
Аноним 23/10/20 Птн 13:35:49 519251350
>>519250
Какую картинку, тупой фантазёр?
Аноним 23/10/20 Птн 13:36:22 519252351
16006524123540.png 4Кб, 170x127
170x127
Аноним 23/10/20 Птн 13:37:24 519253352
>>519252
Ой, тупой опущ, ты чего порвался?
Аноним 23/10/20 Птн 13:39:24 519254353
>>519248
Ну, по крайне мере я не обиженный дурачок с манямирком)
Аноним 23/10/20 Птн 13:42:58 519255354
>>519254
Но к сожалению, - ты дурачок.
Аноним 23/10/20 Птн 15:47:10 519270355
image.png 1137Кб, 1280x914
1280x914
Все мы слышали байку о средней длине пениса в России, кой лежит между 13-ю см и 15-ти сантиметрами счастья. Однако, у меня есть несколько вопросов. Я физик по образованию и учили меня, что данные всегда надо перепроверять. Начал я узнавать у своих знакомых и выяснил, что длинна пениса основной массы людей колеблется между 16-ю и 18-ю сантиметрами, при этом мой пенис 18,5 сантиметров в полностью боевом состоянии. На основание своих наблюдений могу смело выдвинуть гипотезу, о том, что средняя длинна писюна в России составляет 17 сантиметров. Возникает вопрос, каким образом измеряли хуи для составления предыдущей статистики, ни мне, ни моим знакомым никогда не измеряли писюны врачи.
Аноним 23/10/20 Птн 15:53:54 519271356
>>519270
По трупам в морге.
Аноним 23/10/20 Птн 15:55:55 519272357
>>519271
Или меряют новорожденным, а потом складывают с трупами
Аноним 23/10/20 Птн 16:52:58 519274358
>>519246
>Как опровергается? Что за эксперимент?
Неравенства Белла
Аноним 23/10/20 Птн 16:54:07 519275359
>>519246
>Ебать шиза, зачем частице это знать?
Потому что до измерения у нее нет конкретного направления спина. Обе частицы разлетаются в неопределенности. Они "решают" только в момент измерения.
Аноним 23/10/20 Птн 17:03:39 519276360
>>519270
У меня 14 но я никому не давал свой пипидон мерить
Аноним 23/10/20 Птн 17:13:20 519277361
>>519276
Когда подрастешь будет 17-18
Аноним 23/10/20 Птн 17:34:47 519278362
>>519275
>Потому что до измерения у нее нет конкретного направления спина.
Почему?-потому!

Какже нету, если есть? Что доказывает что нету?
Аноним 23/10/20 Птн 17:46:13 519279363
>>519277
Мне 26, говорят мужчины растут до 40 лет. Плюс там метаболизм замедлится, жир появится, а ядрыщ.
Думаешь у меня будет болтяра? Надо только подождать чуть чуть?
Аноним 23/10/20 Птн 17:55:46 519280364
>>519279
> жир появится
Тогда станет короче
Аноним 23/10/20 Птн 18:04:53 519281365
>>519278
>Что доказывает что нету?
>>519274
Неравенства Белла, или другие похожие эксперименты. Если ты совсем гуманитарий, то тебе вот это видео подойдет.
https://youtu.be/zcqZHYo7ONs
Аноним 23/10/20 Птн 19:33:44 519285366
>>519281
Почему они уверены что при эксперименте когда выпускается два фотона которые поведут себя одинаково на большом расстоянии друг от друга, что они выпускают фотоны которые поведут себя одинаково?
Ну это вроде как понял, довольно большая уверенность достигается большим числом проверяющих измерений.

Но всё не могу въехать в это:
Сначала летит ного разных фотонов, но в фильтра расположенные в одном месте. Часть отъсеивается

А, долшло, типа раньше отъсеивались фотоны потому что у них якобы были скрытые параметры.
Но теперь отсеиваются по-разному фотоны которые должны были иметь одинаковые скрытые параметры если бы они были.

Типа колебрующие измерения показывают допустим что прибор выпускает одинаковые фотоны с вероятностью 95% например.
А когда они проходят через комбинацию фильтров отфильтровка не те же 95% например, а те веротяностные 85% и 15% как когда стрелят кучей фотонов без заморачивания с их одинаковостью.

Ну интересно, если это так.

Так выходит это чисто экспериментальная штука, я думал теорема бэлла или что-то такое чисто по логике разъябывает неслучайную природу вселенной.
А она лишь теоретически допускает что можно провести такой эксперимент. Так?


И ещё.. чёт я не врубился, они говорят о том что можно предположить, что это проход через средний третий фильтр влияет на то, как оно пройдёт через последний.
Как от этого уходят в эксперименте?
Т.е. допустим, колебруещие измерения-проход через 2а или 1н фильтр. Они показывают что приблр выдаёт одинаковые фотоны 95% раз.
Далее вставляем 3-й фильтр, и пошёл рандом.

Получается, вроде как, что допустим 2-мя фильтрами показали что прибор выдаёт одинаковые фотоны по 1-му скрытому параметрв, но вот допустим 2-й скрытый параметр, который влияет на то как пройдут через 3-й фильтр по середине...
...хотя.. фильтры же одинаковые
Да и рандом бы на этом фильтре давал 50/50, наверное, а не те 15/85
Ну да, вроде комбинируя можно исключить все "а если...."
Аноним 23/10/20 Птн 20:13:55 519288367
>>519285
>Как от этого уходят в эксперименте?
Делают пару одинаковых фотонов (они реально одинаковые, они полностью ведут себя одинаково, если один пролетит то и другой пролетит, если один не пролетит то и другой не пролетит. Ну если ты возьмешь одинаковые системы линз) разводят их на большое расстояние (чтобы друг друга не могли волновать), и потом уже все "если" проверяют, зная, что они одинаковые.
Аноним 23/10/20 Птн 21:52:23 519291368
что есть по теории вероятности для даунов?
Аноним 23/10/20 Птн 22:49:03 519292369
>>518643
Пиздит жидомасон, не слушайте его.

>>518625
Бозон Хигса позволяет менять реальность в точках бифуркации, вызывая эффект Манделы.
Аноним 23/10/20 Птн 22:50:50 519293370
>>519292
Зато это не пиздит. Кстати, дайте ему пизды
Аноним 23/10/20 Птн 23:20:33 519294371
>>519179
Ньютоновский детерминизм: Мы строим фазовое пространство и задаем в нем поток. Каждая точка переходит в соседнею единственным образов. В итоге получаем единственная возможная эволюция систем из текущего состояния.
В квантовой теории состояния системы не привязаны к скоростям и координатам, а связь между ними делается через специальные процедуры над этим пространством. Эволюция системы происходит уже на уровне квантовых состояний, к которым мы уже не имеем доступа. Из координат и скоростей мы не можем узнать точно в каком состоянии находиться квантовая система и следовательно не можем узнать по какому единственному пути она будет эволюционировать. Вот и тут и рушиться ньютоновский детерминизм из-за принципиальной невозможности получить полную информацию о квантовых состояний.
Впрочем квантовые системы детерминированы в том смысле, что одно квантовое состояние переходит в другое единственным образом. Только все КАК НА САМОМ ДЕЛЕ мы никак не можем увидеть.

А через среднее значение мы только дополнительно избавляемся от лишней информации.
Аноним 23/10/20 Птн 23:25:19 519295372
>>519293
Ты ебучий дегенерат, тебя по=хорошему надо сжечь в печи.
Аноним 23/10/20 Птн 23:27:28 519296373
>>519229
Ты скозал? Доказательства есть, лалка?
Аноним 23/10/20 Птн 23:42:10 519298374
>>519295
У этого точка бифуркации сколлапсировала, несите нового
Аноним 23/10/20 Птн 23:56:31 519299375
>>519247
>Что очевидно? Потому что ты написал "очевидно"? Очевидно нет.
Чиво, блять? Что тебе не очевидно, ёпт?
Если что-то присутствует сейчас, значит оно отсутствовало позже, и это присутствие появилось в процессе появления, а поскольку это процессс, и любой процесс во времени направлен, то само понятие присутствие связано со временем.

И да, какая нафиг частота между событиями? Ты это сам придумал?
И чем тебе постоянная Дирака, гравитационная постоянная, и скорость света в вакууме - не фундаментальные константы?
Вот же формула: https://ru.wikipedia.org/wiki/Планковское_время
Аноним 24/10/20 Суб 00:23:16 519302376
>>519299
>то
Схерали ты решил что это селдует из того, только потому что ты сказал?
Это пустословие, болтология.
Аноним 24/10/20 Суб 00:24:28 519303377
>>519299
>гравитационная постоянная
Не фундаментальная вещь.
Аноним 24/10/20 Суб 00:37:07 519304378
>>519299
>И да, какая нафиг частота между событиями? Ты это сам придумал?
>И чем тебе постоянная Дирака, гравитационная постоянная, и скорость света в вакууме - не фундаментальные константы?
>Вот же формула:
Как связано время и планковское время?

Почему время выводят через какие-то каличгые колебания электрона на "орбите" атома какого-то каличного, а не через фундаментальные постоянные?
Аноним 24/10/20 Суб 01:22:06 519305379
Так а в чём был изначальный проблемес?
Странное поведение света при проходе через фильтры, которое никто не мог объяснить?

И эйнштейн взялся со своей ото/сто пытаться это объяснить и соснул?
Но вроде применимость ото/сто чётко оговорена изначально и это не её область.
Он попытался не ейю это объяснить но всосал?
Или он не пытался, а ему просто не понравилось объясненип которое дала квантовая механика?
Аноним 24/10/20 Суб 04:09:47 519308380
Что скажете про развитие мозга через тренировку на ЭЭГ?

Занимаюсь месяц, есть результат, но мне сложно его измерить.
Аноним 24/10/20 Суб 05:16:12 519309381
>>519308
В чём проявляется результат?
Аноним 24/10/20 Суб 05:53:30 519310382
>>519309
Могу капчу с первого раза отгадать.
Аноним 24/10/20 Суб 06:08:31 519311383
>>519310
Поздравляю, вы не робот.
биоробот
Аноним 24/10/20 Суб 07:08:17 519314384
>>519302
Я тебе толдычу лишь о том, что невозможно, блядь, представить себе,
какое-либо существование чего-либо,
без ВРЕМЕНИ,
то есть, без хотя-бы одного отрезка планковского ВРЕМЕНИ,
в течении КОТОРОГО, это нечто - СУЩЕСТВУЕТ.
То есть, само слово "существование", связано со временем, блядь,
и если времени нет, но не существует вообще нихуя, как в сингулярности той, космологической.

Но ты говоришь "докажи, что время существует".
Я немного переформулирую это, даже.
"Докажи, что интервал времени существует,
В ТЕЧЕНИИ НЕКОЕГО ИНТЕРВАЛА ВРЕМЕНИ,
который существует в течении некоего интервала времени, который... И т. д."
Вот сиди и доказывай эту рекурсию, теперь. Ололо.

>>519303
>>гравитационная постоянная
>Не фундаментальная вещь.
Ну почему-же? https://ru.wikipedia.org/wiki/Фундаментальные_физические_постоянные
Она есть там.

>>519304
>Как связано время и планковское время?
Связана через континуум, через взаимосвязь последовательности отрезков планковского времени, жи.

>>519304
>Почему время выводят через какие-то каличгые колебания электрона на "орбите" атома какого-то >каличного, а не через фундаментальные постоянные?
Хуй знает что там у тебя выводят, через электроны.
Но если ты об атомных часах, то гугли - квантовый дискриминатор.
Там, короче, выбирают такие материалы, которые наиболее высокочастотно и периодично меняют свои состояния,
которые можно успеть зарегистрировать, и уже после подсчёта числа которых,
получают наиболее точный результат измерения времени,
под который уже подгоняют кварцевый резонатор, чтобы не было отклонений даже на микросекунду.
Аноним 24/10/20 Суб 07:09:19 519315385
>>519310
Нейросетки тоже могут. Даже на транзисторах реализованные. Гугли машинное зрение. Ололо.
Аноним 24/10/20 Суб 08:54:50 519320386
>>519315
Так нейросетки сами заливают фотки и проверяют какие ты выбрал. А вы думали этим Абу занимается лично
Аноним 24/10/20 Суб 09:43:36 519321387
>>519309
Начал слышать второй канал одновременно, а не переключаясь между звуками. Сложно описать, но чувствовать.

Расширился угол обзора и могу зрительно удерживать 2-3 объекта одновременно, а если переключаться до 8.

Начал видеть в глубину ( не знаю как эффекти называется. Грубо говоря, если в длинном туннеле, то видно не просто конец туннеля, но и весь путь к нему)

Мозг стал быстрее реагировать на потенциальные халявные места.

Стал точнее считать в уме.

Перестали раздражать многие звуки.

Научился в базовый самоконтроле и начал борьбу с прокрастинацией.

Аноним 24/10/20 Суб 10:53:03 519327388
image.png 687Кб, 625x601
625x601
>>519321
> Начал слышать второй канал одновременно, а не переключаясь между звуками. Сложно описать, но чувствовать.
Аноним 24/10/20 Суб 11:17:55 519329389
Аноним 24/10/20 Суб 11:18:35 519330390
Аноним 24/10/20 Суб 11:21:26 519331391
>>519321
Это реально или только впечатления? Практические, не субьективные, полезные, верифицируемые результаты есть?
В жизни как помогает?

И что значит ээг? Электроэнцефалограмма?
Ты себе мозг электричеством фигачишь?
Аноним 24/10/20 Суб 12:32:39 519334392
>>519321
Похоже на шизоидные маняфантазии.

Знаешь, такие фантазёры которые "чувствуют", а не делают дела.
Нихуя не умеют реального, нихуя не умеют, зато "чувствуют". Фантазируют себе ощущения всякие.
Аноним 24/10/20 Суб 12:35:32 519335393
>>519321
Ебани себя об ЛЭП, базарю, начнешь видеть и слышать каналы всего астрала.
Аноним 24/10/20 Суб 12:35:48 519336394
>>519314
>какое-либо существование чего-либо,
>без ВРЕМЕНИ
Шиза, тебя просто промыли маняфантазией.

Вот смотри, происходят события, каждое с какой-то своей частотой относительно других, допустим покв происходит 1 раз, другое происходит 467436 раз, другое 2, а третье 1.567 раза.
Зачем для этого время?
Аноним 24/10/20 Суб 12:37:08 519337395
>>519314
>Вот сиди и доказывай эту рекурсию, теперь. Ололо.
Зачем мне доказывать маняфантазию которую ТЫ выдумал?
Это работает не так, я просто говорю "этого нет", и вот ты докащывай что есть.
Аноним 24/10/20 Суб 12:38:13 519338396
Аноним 24/10/20 Суб 12:41:27 519339397
>>519314
>Хуй знает что там у тебя выводят, через электроны.
>Но если ты об атомных часах, то гугли - квантовый дискриминатор.
>Там, короче, выбирают такие материалы, которые наиболее высокочастотно и периодично меняют свои состояния,
>которые можно успеть зарегистрировать, и уже после подсчёта числа которых,
>получают наиболее точный результат измерения времени,
>под который уже подгоняют кварцевый резонатор, чтобы не было отклонений даже на микросекунду
Нет, ты немного не понял, тут речь о фундаментальности.

Скорость света тоже раньше измеряли по метру, а теперь метр измерчют по скорости света.

Раз время такая пездатая штука, то почему время измеряют по колкбаниям электрона у атома, а не колебания электрона у атома по времени?

Пока это так-выходит что время-хуйня из под ногтя.
Аноним 24/10/20 Суб 12:53:56 519340398
>>519314
>Связана через континуум, через взаимосвязь последовательности отрезков планковского времени, жи.

Ты разбирался в планковских величинах? Что такое планковская длина?
Свет констаната, ок, зафиксировались, но далее длина.
Минимальная длина, по размерам минимальной чёрной дыры, как-то так.
Но чёрная дыра, вроде да, это фундаментальная штука, т.к. по сути, выражается через свет. По крайней мере вроде эта планковчкая.
Но планковское время...
Если оно выражается как свет, прошедший через "свет", получается какая-то хуйня. Свет выраженный через свет=время.
А может и не хуйня, а приблизились к какой-то реально фундаментальной константе.

Ну ок, там есть ещё гравитационная постоянная, ну чтобы собственно посчитать чёрную дыру.
Но, гравитация по теории относительности полностью завязана на скорости света.

ГрИ вообще я сейчас дошёл, что лучше говорить о пространство-свете, ведь всё есть движение света-волн, через пространство.
Только некоторые волны от движения со скоростью света искривляют пространство и начинают двигаться медленнее относительно остальных. Точнее двигаются так же, но проходят меньше расстояние, т.к. путь больше.
Аноним 24/10/20 Суб 13:07:03 519343399
>>519314

И начнём вообще с того, что не множь блять сущности.
Действия и взаимодействия происходят, происходят относительно
друг-друга, нахера время?

Тебе хочется вывести какой-то коэффициент, чисто для себя, чтобы сравнивать эти события.
Но тут остаётся вопрос с планковским, ок.

Но, время полностью херачит теория относительности. Да, она им оперирует, но она полностью же его и захуячивает.
Если время такая пездатая штука, то как оно может быть относительным?

Хочешь сказать планковское время нельзя исказить?
А как тогда возможно исказить другое, не планковское время?
Аноним 24/10/20 Суб 13:42:21 519344400
0.jpg 7Кб, 480x360
480x360
А что если по проводам пускать не электроны, а антиэлектроны то есть позитроны?
Аноним 24/10/20 Суб 13:46:06 519346401
Аноним 24/10/20 Суб 14:04:05 519347402
>>519344
Кстати, а где все позитроны...
Аноним 24/10/20 Суб 14:21:29 519348403
>>519347
Хороший вопрос...
Аноним 24/10/20 Суб 15:39:09 519353404
>>519347
Умерли как только родились
Аноним 24/10/20 Суб 15:48:12 519354405
>>519353
Но можно массово производить позитроны и запускать по проводам.
Аноним 24/10/20 Суб 16:03:00 519355406
>>519354
Может еще и нейтрино вооружить?
Аноним 24/10/20 Суб 16:39:22 519357407
image.png 5Кб, 386x171
386x171
Сап, саентач, я электрон. Я щас в точке А, хочу в точку Б. Далее у меня развилка, на одном проводе охуенный резистор, который не резистит ничего (r -> 0), на другом - хуевый резистор, который резистит всё (R -> бесконечность). Я, хоть и электрон, но в школе чуть-чуть но видимо не достаточно учился, потому знаю, что я пойду по дороге с r, причём всегда - нахуй мне тратить силы на переход бесконечного сопротивления? Но вот только я вблизи точки А и не вижу, какой резистор где. Однако уже знаю, по какой дороге я пойду. Вопрос - как?
Ну или r<R, но не сильно. Нас теперь 150 штук, и мы заранее делимся на, например, группы по 100 и 50 электронов и где сопротивление меньше - туда народу идёт больше. Откуда мы знаем, как делиться и кому куда идти?
Аноним 24/10/20 Суб 18:01:28 519366408
>>519357
Потому что тебя толкают другие электроны туда.

Ну или электромагнитное поле. Потому что там где пройти легче-будет больше ток, и сильнее электромагнитное поле.
Аноним 24/10/20 Суб 18:03:25 519367409
>>519357
И там нет такой ситуации что вас группа 150 штук, и вы думаете куда пройти, вы равномерно распределены по проводам, а не стоите в проходе.
Т.е. когда начнётся движение, в месте с большим сопротивлением сразу образуется толпучка, и те кто подальше в проходе стоит пойдут туда где свободнее.

Аноним 24/10/20 Суб 18:07:24 519369410
>>519366
Ну ток будет больше как раз за счёт таких как я, кто выбрал путь полегче. А поле - разве при параллельном соединении напряжения не равны? Напряжение~потенциал~поле. Где я не прав?
>>519367
Т.е. в условный момент включения тока я могу пойти куда угодно 50/50%, но "на второй круг" из-за "толкучки" вероятней пойду туда, где сопротивление меньше?
Аноним 24/10/20 Суб 18:08:01 519371411
>>519369
>включения тока
*поля/напряжения, конечно
Аноним 24/10/20 Суб 18:39:51 519373412
>>519369
>Т.е. в условный момент включения тока я могу пойти куда угодно 50/50%, но "на второй круг" из-за "толкучки" вероятней пойду туда, где сопротивление меньше?
Нет, с чего это?
Тут смотря где ты находишься, если где-то на развилке, то пойдёшь с вероятностью пропорционально сопротивлений.
Аноним 24/10/20 Суб 18:42:28 519375413
>>519373
Ну так вопрос - чому так? Я "вижу" и "рассчитываю" сопротивления впереди?
Аноним 24/10/20 Суб 18:56:48 519376414
>>519375
Я ж говорю тебя поток увлекает в нужную сторону.
Рассматривать эоо можно разными моделями, я тебе уже говорил, или другие электроны пихают, или поле сильнее там где надо.

Ты сказал что "напряжение одинаковое", но оно не одинаковое, если движение именно началось, смотри "падение напряжения".
Аноним 24/10/20 Суб 19:13:54 519379415
Все знают про большой взрыв (что-то ебануло и появилось всё), а как насчет второго взрыва по величине, моченые что-то регистрировали, или теории есть?
Аноним 24/10/20 Суб 19:16:55 519380416
>>519379
Взрыва не было, было расширение.

Саиый большой "второй взрыв" который может быть-это слияние двух сверхмассивных чёрных дыр, но там энергия выделяется в виде гравитационных волн.
Аноним 24/10/20 Суб 19:17:34 519381417
>>519376
>но оно не одинаковое, если движение именно началось
Понял-принял, спасибо.
Аноним 24/10/20 Суб 19:25:23 519382418
>>519380
Всего лишь гравитационные волны, слабенько
Аноним 24/10/20 Суб 19:39:35 519384419
>>519382
я все охереваю с мочёных,как они их определяют эти волны?
Аноним 24/10/20 Суб 19:52:34 519385420
>>519384
Регистрируя сжатие пространства.
Аноним 24/10/20 Суб 19:52:52 519386421
Аноним 24/10/20 Суб 19:54:12 519387422
>>519385
еще лучше,а растяжение пространсва как?
Аноним 24/10/20 Суб 19:58:26 519388423
>>519387
А сжатие никогда вроде не регестрировали.
Аноним 24/10/20 Суб 22:48:28 519406424
>>519387
пускают лазер на несколько километров и засекают время. Скорость света мы знаем. Профит!
Аноним 25/10/20 Вск 00:24:09 519415425
>>519384
Нет никаких гравитационных волн. Гравитация потенциальная функция. Для процессов, описываемых такими функциями, не свойственны колебания, так как нет внутренней взаимосвязи роста и падения показателя процесса. От изменяется только с изменением значения возбудителя потенциала. Изменение потенциальной функции в этом случае называют смещением. В случае гравитации: есть изменение тяготеющей массы - есть смещение гравитационного потенциала.
Есть представление о дефекте массы как процессе, сопровождаемом спадом гравитации. Ослабление гравитационного поля, его потенциал, будет распространяться со свойственной ему скоростью. Но чтобы была волна в обычном понимании, нужны колебания: подъемы и спады, спады и подъемы.
Что же заставляет гравитацию возрастать, подниматься и опять спадать, образуя волну?
Пока нет хотя бы гипотетических предположений такого явления, гравитационные волны остаются математическим вывертом.
Разве что предположить, что существует некая гравитационная «упругость» пространства. Она может проистекать, скажем, из взаимодействия барионного тяготения с отталкивающей темной энергией. Тогда следует искать характеристический параметр, добротность, собственные колебания, резонансные явления.
А может все дело в источнике: процесс дефекта массы происходит не одномоментно, а ступенчато. Ступенчато пойдет и спад гравитации. Тогда представления о колебаниях явятся следствием свойств приборов – дифференциально-разностные схемы измерения обычно убирают постоянную и медленную составляющие, превращая «ступеньки» в «забор», т.е. однонаправленную последовательность изменений в колебательную.
В любом случае, пока нет суждений об эффекте роста гравитации вслед за ее падением, не может быть представлений о явлении гравитационных волн.
Аноним 25/10/20 Вск 02:53:55 519420426
https://www.live-and-learn.ru/catalog/article/prichina-nizkoy-rabotosposobnosti/

Это правда, что причина низкой работоспособности в неспособности переключиться с левого полушария на правое?

Получается, если развить каким-то образом правое полушарие, то робоспособность увеличиться?
Аноним 25/10/20 Вск 03:19:44 519424427
Если мир состоит из 6 кварков и 6 лептонов которые объединены в 3 поколения, то как это получилось?
Аноним 25/10/20 Вск 03:20:43 519425428
Аноним 25/10/20 Вск 04:18:23 519426429
16032016325670.jpg 43Кб, 480x360
480x360
Аноним 25/10/20 Вск 04:58:28 519427430
>>519415
Ты чё, конч? Какой показатель процесса? Какие подъёмы-спады, какие ступеньки-заборы? Ты вообще в курсе, что волны - это не на поверхности воды?

Про ОТО точно не в курсе, но это ладно.

А электромагнетизм вообще описывается потенциалом, тем не менее, электромагнитные волны вполне есть.
Аноним 25/10/20 Вск 07:12:36 519430431
>>519426
Ну, каждой твари по паре. слышал о таком?
Аноним 25/10/20 Вск 08:51:40 519440432
Аноним 25/10/20 Вск 08:54:01 519442433
>>519424
Особенность электрослабого взаимодействия. Которая вытекает из свойств группы SU(2), на которой строится взаимодействие. (На самом деле SU(2)xU(1))
Аноним 25/10/20 Вск 09:15:21 519445434
>>519424
Не состоит.Это прост так сказали или подогнали факты
Аноним 25/10/20 Вск 11:10:57 519447435
Слышьте, учёные в говне мочёные, а ну-ка быра объясните мне что такое интеллект и как его развить, во!
Аноним 25/10/20 Вск 11:45:31 519454436
>>519447
>что такое интеллект
Умение оперировать эффективно разными структурами, понимать как оно так все устроено.

>как развить
У каждого есть свои гены и своя предрасположенность, просто надо продолжать заниматься в чем разбираешься, придет опыта интеллект будет на том же уровне..
Аноним 25/10/20 Вск 11:47:42 519456437
почему реликтовое излучение только на самом краю мы видим?
Аноним 25/10/20 Вск 12:06:57 519462438
>>519447
Способность поставить и решать различные задачи различными способами. Задачи могут быть разнопрофильными от поиск пути в пятерочку до выстаивание модели поведение других субъектов. Чем интеллект круче, тем субъект быстрее и эффективнее справляется с этим говном.
А развивать интеллект можно только одним способом: постоянно решать разнородные задачи. Связи в мозгу сами собой не появятся.
Аноним 25/10/20 Вск 12:09:23 519463439
>>519462
спасебо, хули.
>постоянно решать разнородные задачи
а где искать?
Математическое мышление так же кочается? Там ведь опыт не нужен, творческое мышление и всё такое
Аноним 25/10/20 Вск 12:29:27 519469440
>>519463
Если ты не конченный безвольный двачер, то тебе жизнь сама постоянно будет подкидывать различные задачи.
Банальное общение с большим количеством людей сильно бустить интеллект, потому что тебе надо владеть языковыми конструкциями и прогнозировать поведение других людей.
И нет математическое мышления. Задачи в математики в отличие от других задач отличаются только уровнем абстракции и некоторых медскиллзов.
Для реального прокачивание интеллекта нужно вкатываться в незнакомые профессии и получать всякие медскиллзы.
Аноним 25/10/20 Вск 14:03:40 519476441
>>519469
>Банальное общение с большим количеством людей сильно бустить интеллект, потому что тебе надо владеть языковыми конструкциями и прогнозировать поведение других людей.
вот только этим я уже и так занимаюсь сам по себе
>И нет математическое мышления. Задачи в математики в отличие от других задач отличаются только уровнем абстракции и некоторых медскиллзов.
хер его знает, где-то это обозвали математическим мышлением. Тут вопрос в том, прокачивается ли это умение доказывать теоремы и задачи, ведь опыт в этом накопить невозможно, каждый раз задача новая.
Аноним 25/10/20 Вск 17:38:13 519499442
16027074362521.png 65Кб, 180x178
180x178
>>517798 (OP)
Вопрос специально для треда тупых вопросов.
Может кто посоветовать книг по экономике так скажем для очень тупых? Я чувствую себя отсталым дегенератом, который не понимает значения "инфляции" даже с википедией, а не говоря уже о всяких санкциях-хуянкциях и нахуй мы вообще отправляем гуманитарку во всякие Нигерии.
Аноним 25/10/20 Вск 18:30:03 519505443
>>519499
Учебник экономики за 8-9 класс
Аноним 25/10/20 Вск 18:32:08 519506444
image.png 493Кб, 471x465
471x465
Аноним 25/10/20 Вск 18:38:36 519507445
1603640314592.jpg 23Кб, 720x302
720x302
Почему не производят квадрокоптеры больших размеров, если небольшие дроны зарекомендовали себя лучше, чем одновинтовые вертолёты? Только конкретно, а не "пок пок пок плохо масштабируются".
Аноним 25/10/20 Вск 18:39:26 519508446
Опа!.webm 802Кб, 640x360, 00:00:07
640x360
>>519505
>экономика
>8-9 класс
Прошлое, которое я упустил
Будущее, до которого я еще не дожил
Пиздец
Аноним 25/10/20 Вск 18:40:31 519509447
>>519507
>Почему не производят квадрокоптеры больших размеров
а нахуй он нужон, этот ваш гигакоптер?
Аноним 25/10/20 Вск 18:58:44 519513448
>>519507
>Почему не производят квадрокоптеры больших размеров
Потому что пок-пок плохо масштабируются прочности материалов, кудах, конечно же. Если в масштабе пары метров твой пикрелейтед может повернуть винты в разные стороны и его не разорвёт, то с большим коптером это не прокатит. Самолёты, няша, делаются из толстой фольги, и никаких сложных и странных нагрузок они не терпят.
Аноним 25/10/20 Вск 19:09:01 519514449
Аноним 25/10/20 Вск 19:51:22 519517450
>>519507

Потому что закон квадрата-куба.

Он определяет все формы всех структур.

Аноним 25/10/20 Вск 19:56:47 519518451
>>519499
>отправляем гуманитарку во всякие Нигерии
Потому что пидарашки опущенные шлюхи которые оплатят пыни все его игры.
Аноним 25/10/20 Вск 20:02:39 519519452
>>519517
Большие машинки не сильно дороже маленьких
Аноним 25/10/20 Вск 20:03:05 519520453
16036410379950.jpg 375Кб, 1433x2862
1433x2862
>>519518
> пидарашки опущенные шлюхи которые оплатят пыни все его игры
Аноним 25/10/20 Вск 20:38:47 519522454
1603647526317.png 962Кб, 1518x1776
1518x1776
1603647526446.png 1175Кб, 1518x1776
1518x1776
>>519513
> Потому что пок-пок плохо масштабируются прочности материалов, кудах, конечно же. Если в масштабе пары метров твой пикрелейтед может повернуть винты в разные стороны и его не разорвёт, то с большим коптером это не прокатит.
>>519517
> Потому что закон квадрата-куба.

Да нихуя, прочность скейлится абсолютно линейно. Вот я даже для вас кукаретиков посчитал максимальные напряжения в одном образце и во втором, который увеличил в три раза. Напряжения тоже линейно увеличились в три раза. Домашние квадрокоптеры делаются из папье-маше, пластика, китайского люминия и небольшого количества углеволокна. Нормальные вертолеты делаются из нормальных материалов, которые свободно те нагрузки примут на четыре оси, как свободно их принимают на одну ось.

Причина должна быть в другом, не в прочности.
Аноним 25/10/20 Вск 21:46:08 519525455
>>519520
Пидарашья шлюха, ты чего порвалась?
Аноним 25/10/20 Вск 21:48:15 519526456
>>519522
>Да нихуя, прочность скейлится абсолютно линейно.

Бля, дебил, нахуя ты кукарекаешь если даже не можешь хотя бы википедию почитать?

Гугли блять закон квадрата-куба.

Или ещё лучше иди и сбросся с крыши вместе с муравьём.
Муравью ничего не будет из-за падения, а значит и тебе тоже.
Аноним 25/10/20 Вск 21:52:53 519527457
>>519526
Какой же ты дегенерат, господи, и такие ещё советы дают в треде. То что объем и соответственно масса меняется в кубе это даже соседская собака знает. А прочность материала определяется максимально допустимым напряжением, а напряжения меняются линейно даже при увеличении массы, сука ты тупая. Харкнул тебе в рожу, не лезь блядь туда куда не знаешь.
Аноним 25/10/20 Вск 21:59:49 519528458
>>519525
вы хуесос, вам в /b
Аноним 25/10/20 Вск 23:48:30 519533459
>>519528
Пидарашья шлюха, ты чего порвалась?
Аноним 25/10/20 Вск 23:49:53 519535460
>>519527
>даже соседская собака знает
Ты выходит тупее собаки?
Аноним 26/10/20 Пнд 00:04:47 519536461
>>519535
Че понян как обосрался и решил съехать на тупости? Типичное аиб животное, дада ты.
Аноним 26/10/20 Пнд 00:39:49 519541462
>>519536
Где я обосрался? Что за маняфантазии тупой собаки?
Аноним 26/10/20 Пнд 00:45:35 519542463
>>519541
Чмоша тебе уже объяснили где и с пруфами, перестань срать под себя.
Аноним 26/10/20 Пнд 02:29:13 519547464
>>519542
Какие пруфы? Твои маняфантазии и бредни тупой собаки это не пруфы.
Аноним 26/10/20 Пнд 08:47:11 519552465
think.jpg 14Кб, 112x112
112x112
Я вот задумался, почему пердеж и говно воняют по разному?
Пердеж еще более менее терпимо, а некоторым даже нравится нюхать. Говно же всегда воняет шопездец.
Аноним 26/10/20 Пнд 09:27:06 519553466
>>519552
Пердеж это запах непереваренных продуктов.
Говно это уже готовый продукт переработки.
Аноним 26/10/20 Пнд 09:32:57 519554467
>>519553
Гастрономия какая-то.
Аноним 26/10/20 Пнд 11:49:53 519565468
Вот смотрю я например мат.лекции, допустим по этальным когомологиям. Стоит выйти чуть дальше мат.анализа - все куда-то сливаются. Почему эти сравнительно простые топики набирают по 5-10 тыс просмотров в лучшем случае? Если подвести вопрос более конкретно: я заблуждаюсь или среди 7 млрд людей так мало умников? Начинаю верить в теорию 10^9 или даже 10^8.
Аноним 26/10/20 Пнд 12:07:49 519566469
>>519565
Ну смотри, ты родился в семье нищебродов. В стране с хуевой экономикой. У тебя дефицит необходимый для жизни вещей: места, еды, любьми-уважения и всякое такое. И все заботы как так подрастаешь вертсят вокруг хуярить на смену чтобы оплатить проезд коммуналку и еду чтобы продолжать хуярить на смену. Сил не остается даже на подрочить. Какая нахуй этальная когомология в такой ситуации вася. Такое позволить себе может только богема, мажорчики, нахлебники и прочие фартовые блатные ребяты. Думаешь мало кому хотелось бы сидеть и лампово изучать структуру вселенной? Почитывать книжонки за пледиком? Решать задачки на спор?
на земле среди 7 млрд не мало умников. На земле среди 7 млрд мало успешных людей, которым не нужно выживать.

Аноним 26/10/20 Пнд 12:23:17 519568470
>>519566
>Думаешь мало кому хотелось бы сидеть и лампово изучать структуру вселенной? Почитывать книжонки за пледиком? Решать задачки на спор?
Если ты хоть раз жил в коммуналке с "рабочим классом" или ездил в плацкарте с дембелями, то ты знаешь ответ. И ответ да, мало кому. Они свободное время потратят так же бездарно. Нет, ну скорее всего среди них будет 1-2%, кому это будет интересно. Но остальные 98% выберут какую-нибудь более отупляющую хуйню. Это отчетливо можно проследить по судьбам людей, которые выигрывали миллионы в лотереях.
Аноним 26/10/20 Пнд 12:41:10 519569471
>>519568
>знаешь ответ
Не принижай человеческий потенциал.
Все эти люди родились в определенных условиях от таких же людей, всего того о чем я говорил. Их поведение это способ выжить в их среде, не сойти с ума и не выпилиться.

>миллионы в лотереях
У тебя шикарная логика(нет). Просто апплодирую как кто-то может так мастерски демагогично натягивать сову на глобус.
Очевидно, люди, победившие в лотереях, про которых ты говоришь, не побеждали в процессе рождения. Они получили эту награду, которая в их ситуации совсем не награда совсем не в благосклонном к саморазвитию состоянии. Шальные деньги, вот это вот все. Их никто не учил, никто не показывал. Точнее учили-то их другому. Не говоря о том что в таких ситуациях сильно плющит голову, очень мощное потрясение, изменение эмоционального уровня. Полностью неприспособленную особось оместить в абсолютно неизвестную среду-кукуха от такого не поедет только у очень сильных духов, нервно стабильных. Стоит ли говорить, что стабильность нервной системы никак не коррелирует с интеллектом.
Ты бы также сошел с ума. Мб даже похуже чем они.
Аноним 26/10/20 Пнд 12:57:40 519571472
>>519569
>Ты бы также сошел с ума. Мб даже похуже чем они.
Почему? Я в принципе не нуждался никогда в чем-то. За меня даже мою комнату убирает тетя, которую нанимают мои родители.
Аноним 26/10/20 Пнд 13:06:30 519572473
>>519569
>Не принижай человеческий потенциал.
А я потенциал как раз и не принижаю. Знаю что они могут. Но это будет им просто неинтересно. Большинству моих знакомых (из благополучных семей), любая учеба, особенно математика, была в тягость. У них от неё "болела голова". Биология очень четко указывает на всех уровнях, что человек будет стремиться к безделью и лени, это вшито в нашу биохимию на уровне мозга, вшито в каждую клетку. Как раз те люди, которых это прикалывает, решать задачки и спорить о них, как раз эти люди имеют небольшое отклонение. Нет, это отклонение не дает им способностей больше чем у других, это отклонение просто нарушает их центр удовольствий. Мазохистам нравится когда им делают больно, людям решающие задачки нравится, когда их мозг напрягается, даже когда нет рядом опасности и благополучная среда. Это просто небольшое отклонение, которое коррелирует с шизофренией например. Надо быть идеалистом, чтобы верить в Tabula rasa. В 17-0м 19-ом веке может это и было прогрессивно, верить, что среда может слепить из любого что угодно, и что из каждого можно создать философа. Сейчас слишком много данных, указывающих на другое. Хотя конечно же, отрицать что среда влияет на процент людей, которые захотят этим заниматься я не стану, но даже в самых идеальных условиях их будет меньшинство. Большая часть выберет самые легкие удовольствия, остальная часть, которая любит напрягать мозг, выберет видеоигры, и только крайне малая часть сможет уклониться от видеоигр и выбрать математику.
Аноним 26/10/20 Пнд 13:07:11 519573474
>>519571
Фейспалм. Я об этом и говорю. Идеальное олицетворение сытый голодного не разумеет. Ты никогда не нуждался, сформировался в этих условиях и конечно тебе проще с них находится. Ты обучен этому с детства. А тут просто ~30летний реднек выигрывает тонну бабла в ввжих.
Аноним 26/10/20 Пнд 13:13:54 519574475
>>519572
>безедлью и лени
Корректнее было бы сказать к 'к состоянию более подходящему на данный момент для эффективного функционирования организма'.

>отклонение
Это отклонение обычная форма закрепленной сложной привычки. Если научить этому каждого. то сразу дело у всех пойдет как по маслу.

>разделять видеоигры и математику
Ну я уже конечно это понял, но у тебя какое-то мозщное предубежденческое понимание вещей. Игры бывают разные, биология у всех разная - если подать математическую информацию в интересном, игровом ключе, подходящим для определенного типа людей - я думаю, эти люди смогут щелкть математические задачки не хуже 'стереотипных' книжных червей, если даже не лучше(вопрос восприятия и психологии)
Аноним 26/10/20 Пнд 13:37:47 519575476
С каких книг/источников порекомендуете начать изучать биологию в целом? Чтобы понятным языком было написано.
inb4 любой школьный учебник
Аноним 26/10/20 Пнд 13:40:26 519576477
>>519575
Сам изучай, без книг.
Аноним 26/10/20 Пнд 13:50:04 519577478
>>519576
В смысле? Эмпирическим путем что ли? Теорию откуда-то же надо брать.
Аноним 26/10/20 Пнд 13:51:34 519578479
>>519577
Вот сам и сделаешь себе теорию. Будешь всем на ебала ссать, потому что добился всего сам.
Аноним 26/10/20 Пнд 14:54:32 519579480
>>517866
А в южной корее РКНятся наоборот из - за тяжелой работы
Аноним 26/10/20 Пнд 17:33:10 519585481
Поясните за эффективность по Парето. Как я понял, это некая закономерность, при которой достигается наилучшее состояние системы, где соблюдено наиболее оптимальное "равновесие"? Например есть здание, мы его строим, чтобы соблюсти его надёжность, долговечность и при этом его строительство принесло нам прибыль. Вот такое решение, когда оно будет надёжным, обеспечит весь срок эксплуатации и принесёт при этом максимально возможную прибыль и есть то состояние максимальной эффективности по Парето?
Аноним 26/10/20 Пнд 17:35:19 519586482
>>519578
Первое правило эффективности по Торетто - не повторяться в познании бытия.
Аноним 26/10/20 Пнд 17:46:30 519587483
>>519585
Нет. Это приценочное правило, не точное и не универсальное, которое говорит что только часть усилий полезна, а остальное это пинание хуев.
На самом деле это не совсем так так как иногда пинание хуев на самом деле полезно.
Аноним 26/10/20 Пнд 18:58:01 519597484
Бритвой Оккама всегда надо пользоваться или таки есть ситуации, когда более сложное объяснение > легкого? Если последнее, то можно примеров?
Аноним 26/10/20 Пнд 19:17:22 519598485
>>519597
Ну, бля, чето там вращается, поворачивается в обе стороны спиной.
Аноним 26/10/20 Пнд 19:22:21 519599486
>>519597
когда проходишь тест на айкью то надо искать сложные ответы
Аноним 26/10/20 Пнд 22:37:40 519605487
>>519597
Тут смысл в том, что простое решение должно работать.
Решаешь самым простым, если оно не работает или начинаешь усложнять, ты наусложнял, но если появляется более простое, его принимаешь, или сразу другое простое искать.
Если оно перестаёт работать можно к сложному вернуться, если оно будет работать после едн6ё бОльшего усложнения.

Тут главный смысл в работоспособности.

А из двух работающих нужно выбирать то что попроще.
Аноним 27/10/20 Втр 12:41:04 519624488
Детский вопрос химикам. Зачем споласкивать рот после чистки зубов водой ? Ведь зубная паста, так понимаю, не то что не ядовита, а вообще плохо в реакции вступает с желудочным сококм и т.п. Т. е. её нет смысла смывать. Она же будет дейстовать хоть весь день, пока на зубах остаётся. Разве не так ?
Аноним 27/10/20 Втр 12:47:22 519626489
>>519624
Чтобы зубной налет смыть вместе с пастой.
Аноним 27/10/20 Втр 14:27:32 519630490
>>519626
А, я не подумал что пасты механического воздействия: вспениваются чтобы подобрать бактерий с зубов. Думал они все "антибактериальные" т. е. химически травят бактерий.
Аноним 27/10/20 Втр 14:37:46 519631491
У меня дома стоит плотно закупоренная пятилитровая бутылка от воды, наполовину наполненная яблочным соком.
За 4-5 недель в ней накопилось давление. Пятилитровик принял форму яйца. Рифлёный пластик стал идеально гладким. Крышка, сделанная из толстой пластмассы, выгнулась дугой. Когда трогал его неделю назад - был как камень. Теперь трогать боюсь. По ощущениям там атмосфер 6, если не все 8.
Внимание вопрос:
Ебанёт?
Или в какой-то момент система придёт к равновесию и газ просто не сможет больше выделяться, поскольку давление в свободной части пятилитровика больше не допустит новые молекулы, поскольку не будет градиента, не будет разницы между давлением снизу и сверху?
Аноним 27/10/20 Втр 15:47:06 519633492
>>519587
Ты путаешь принцип Парето, так любимый научпоперами, и эффективность Парето, которая является одним из ключевых понятий современной социальной оптимизации.
Впрочем, от /sci/ ничего другого и не ожидаешь

>>519585
Попробуй почитать https://en.wikipedia.org/wiki/Edgeworth_box, а потом погугли какие-нибудь МИТ лекции, например https://ocw.mit.edu/courses/economics/14-03-microeconomic-theory-and-public-policy-fall-2016/lecture-notes/MIT14_03F16_lec10.pdf

>>519597
Главный вопрос - зачем. Если тебе более сложная теория что-то даёт, то пожалуйста. Другое дело, что если моя более простая теория даёт такие же результаты, то может ты зря городишь огород.
Вот например - в традиционной интерпретации квантовой механики понятие траектории для достаточно малых бессмысленно. В то же время бомовская пилотная теория более "сложная" (в смысле, больше адхок придумок), но зато там можно говорить о родных, любимых, интуитивных траекториях. То есть здесь ты решаешь сам - или по пизде идёт интуиция, но теория проще, или более "сложная" теория, но и легко интерпретируемая. Как известно, интерпретации КМ нефизичны, и без понятия траектории развита самая точная в мире наука (КЭД) и работают твои смартфоны и пека.
Аноним 27/10/20 Втр 16:26:19 519634493
>>519631
>система придёт к равновесию и газ просто не сможет больше выделяться,
Зис.
Углекислый газ(а это конечный продукт всяких брожений) хорошо растворяется в воде. И растворимость растет при давлении. При комнатной температуре предел давлений около 3 атмосфер. При повышении давления газ будет уходить в воду.
Аноним 27/10/20 Втр 18:47:33 519638494
>>519336>>519337>>519338>>519339>>519340>>519343
Бля, посоны... Хуя вы понаписывали.
У меня нет времени исследовать всё это, ибо мне надо ддосить вавесы-говнявесы.
Но давайте разберём.

>Шиза, тебя просто промыли маняфантазией.
О какой маняфантазии речь?
Представь себе, что время - это пространственное измерение. Четвёртое. Да, это трудно представить, но ты представь.
Теперь представь, что Вселенная - это четырехмерный объект.
Статичный и неизменный объект. Как трехмерный, но четырехмерный. И весь контент, внутри измерений объекта, он просо ЕСТЬ, но он не СУЩЕСТВУЕТ.
И сам объект не существует. Он просто ЕСТЬ, вне времени,
потому что время - это его компонента.
Да, он может существовать в другом времени, каком-то. Стивен Хокинг писал о комплексном времени, и возможном сущестововании времени, как-бэ перпендикулярном оси нашего времени. Ололо. Тогда, комплексное время, как-бы на комплексной плоскости где-то может исчисляться.
А если есть две оси, то есть как-бы два времени.
Ну так вот, если какое-либо дополнительное время есть, то СУЩЕСТВОВАНИЕ в НЁМ, по мере ТЕЧЕНИЯ этого ВРЕМЕНИ, возможно. Иначе, объект, просто ЕСТЬ, но он не СУЩЕСТВУЕТ, в течении хотя-бы одного отрезка планковского времени. ЛОЛ.
Вот что я имею в виду, когда говорю, что термин "существование" "чего-либо", подразумевает наличие самого времени, в течении которого это "что-либо", таки СУЩЕСТВУЕТ. Поэтому, и не совсем корректно говорить о СУЩЕСТВОВАНИИ самого времени, оно просто ЕСТЬ и всё.

>Вот смотри, происходят события, каждое с какой-то своей частотой относительно других, допустим покв происходит 1 раз, другое происходит 467436 раз, другое 2, а третье 1.567 раза.
>Зачем для этого время?
Так они же во времени и происходят, как они могут происходить, если, блядь - времени нет?

>>519337
>Зачем мне доказывать маняфантазию которую ТЫ выдумал?
Я не просто от балды выдумал. Я ещё и аргументировал. Так что это не совсем маняфантазия.
А доказывать зачем? Ну, чтобы ты понел.
>Это работает не так, я просто говорю "этого нет", и вот ты докащывай что есть.
Это работает не так. Ты реквестируешь доказательство, а анон думает, соизволить ли ему попытаться доказать ЭТО, или же не соизволить.
И если там, рили, нехуй доказывать, то следовательно - и доказывать нехуй. Ололо.

>>519338
>Не тянет она на фундаментальную постоянную, это лишь следствие. Чего-то.
Ну, просто там, черным по белому же: "Фундаментальные_физические_постоянные".
Может ты и прав, и в какой-нибудь теории петлевой квантовой гравитации, или в теории струн, или ещё какой "теории Всего", где гравитация описывается через другие взаимодействия, через тот же хиггсовский механизм, возможно там гравитационная постоянная выводится из других фундаментальных констант. Тогда, на деле, их может быть ещё меньше, а гравитационная постоянная - как-бы составная константа, из этих констант-компонент. Как та же, постоянная Дирака, она связана не только с фундаментальными физическими постоянными, но и с математической константой - с числом пи. А их там тоже немало: https://ru.wikipedia.org/wiki/Математическая_константа

>>519340
>Ты разбирался в планковских величинах? Что такое планковская длина?
Я понимаю так. Пространство-время - четырехмерно и дискретно, и состоит из атомарных частиц - планковских объемов, которые двигаются во времени, состоящем отрезков "планковского времени".
Весь трехмерный объем Вселенной (представь что это куб, но это не куб, блядь),
он как-бы копируется в новый объем (второй куб, рядом с тем кубом, появляется, типа),
и так каждый раз на каждом отрезке "планковского времени".
При этом, внутри каждого из объемов (кубов), Вселенная статична,
а при копировании объема, её состояния могут отличаться.
И именно при изменении состояния, по мере течения времени, возможно развитие Вселенной, и течение различных процессов в ней. Иначе, она была бы статичной и неподвижной,
и объекты, находящиеся в ней, не могли бы двигаться в пространстве вообще.
Вот, глянь все апории Зенона: https://ru.wikipedia.org/wiki/Апории_Зенона#Содержание_апорий_о_движении

Время, в теории относительности не совсем уж и относительно. Оно может только замедляться, а не ускоряться. Относительность, это когда ты летишь на ракете, а тебе кажется, что ты неподвижен, и что это пространство, само летит назад, со скоростью ракеты. Но время назад не пиздует, оно пиздует только вперёд. Относительность проявляется только в одновременности, потому что время может только замедляться, а не ускоряться, и связано это замедление либо с релятивистским, либо с гравитационным замедлением времени. В пределе, до отделения гравитации, в момент планковской эпохи, она полностью компенсировала время, и зажимала его в ноль, внутри космологической сингулярности, как в той же чёрной дыре. Гугли гравитационное замедление времени.
И вот сам этот процесс, если тщательно исследовать его описания, он может показать, как именно время связано с гравитационной постоянной, с гравитацией вцелом, и как именно гравитация (точнее её отделение) - породила это время, и было ли это реально отделением её или же чем-то иным...
Но было бы интересно узнать, существует ли гравитация, или же это что-то составное в каком-то многомерии... Хз. Например, это может быть какое-нить - гравимагнитное взаимодействие...
Вообще, я думаю, стоило бы отдельный тред запилить, и там вариться, по хардкору уже, а не здесь.
Здесь всё смоет скоро.

>>519343
>И начнём вообще с того, что не множь блять сущности.
>Действия и взаимодействия происходят, происходят относительно
>друг-друга, нахера время?
Так они не могут происходить без времени, блядь, как ты этого не поймёшь?
Представь тупо трехмерное пространство. Вот есть объект в нём, скажем - дерево.
Как дерево может пошевелить листиком, если времени нет? Никак.
>Тебе хочется вывести какой-то коэффициент, чисто для себя, чтобы сравнивать эти события.
>Но тут остаётся вопрос с планковским, ок.
Я говорю тебе, что время фундаментально, блядь. Процесс его течения - процесс смены отрезков "планковского времени",
который, как-бэ - "запущен был", изначально.

>Если время такая пездатая штука, то как оно может быть относительным?
Весь прикол, что оно с пространством - завязано в континуум. И при деформации его, искривляется не только пространство, но и время это, с пространством связанное - оно, в результате, замедляться может.

Но, как ты сам, верно подметил - всё это на макромасштабах, а планковское время настолько миллипиздрическое, что ему поебать на все эти эффекты. Походу, оно было искажено лишь раз, в момент появления планковской эпохи. Потому что там такая пездатая гравитация была, гравитация всей вселенной, нахой.
Аноним 27/10/20 Втр 20:37:30 519639495
>>519638
От того что ты сказал "аряяя, существует!!1" и повторил много-много раз аргументом это не стало.
Это лишь твоя маняфантазия.
Аноним 27/10/20 Втр 20:38:49 519640496
>>519638
>Я понимаю так.
Я не спрашивал как ты понимаешь, лол, я спросил ты разобралася?

И дальше пошли опять же маняфантазии.

Ты не фантазируй, ты разбирайся.
Аноним 27/10/20 Втр 20:40:17 519641497
>>519638
>Так они не могут происходить без времени
Твоя маняфантазия.
Понимаешь, от того что ты говоришь "я себе так нафантазировал" это не становится так.
Аноним 27/10/20 Втр 20:41:38 519642498
>>519638
>Я говорю тебе, что время фундаментально, блядь. Процесс его течения - процесс смены отрезков "планковского времени",
Твоя маняфантазия.
Ты не разобрался что такое планковская длина.
Аноним 27/10/20 Втр 20:42:20 519643499
>>519638
>Как дерево может пошевелить листиком, если времени нет? Никак.
Просто берёт и шевелет, потому что происходят взаимодействия атомов.
Аноним 27/10/20 Втр 20:44:17 519644500
>>519638
>Весь прикол, что оно с пространством - завязано в континуум.
Время ни с чем не завязано, с пространством завязана скорость распространения волн в нём.
Нет "пространства-времени", есть "пространство-свет".

А то что в формулке эйнштейна t, так это просто коэффициент для упрощения.
Аноним 27/10/20 Втр 23:26:49 519651501
Думал в un будет тред по наукам, оказалось там школьники, наверно я сюда пришел, интересует биология и все что с ней связано, начал с трехтомника по канону, дальше мне уже нужна программа развития и направлений.

Сам я разработчик с 5 летним стажем и мне все это доебало, хочу магу по биологии получить, благо давно интересовался, а сейчас начал рассматривать как серьезную альтерантиву, учитывая что удалось накопить неплохую фин подушку.
Подскажите с каким essential по биологии ознакомиться, потому что результатов рекурсивного поиска по гуглу дает самые разнообразные варианты
Аноним 27/10/20 Втр 23:31:06 519652502
Добра вам, господа. Суть токова - надо сделать несколько (на самом деле целую кучу) рисунков по спектроскопии.
Хотел ебануть в поверпоинте, но оказалось что некоторые вещи в нем реализовать не так то просто.

Посоветуйте каких-нибудь программ для рисования схем (чертежев), чтоб можно было функцию морзе захерачить не поточечно строя ее в экселе и все в таком духе.
Аноним 28/10/20 Срд 00:14:22 519654503
>>519652
В свое время я ебашил схемы на маткаде, благо есть модули.
А так фотошоп или автокад.
Аноним 28/10/20 Срд 01:02:33 519657504
image.png 50Кб, 482x214
482x214
Вот есть зазор (вал по диаметру меньше чем дырка, в которую его вставляют)
а есть натяг (вал по диаметру больше чем дырка, в которую его вставляют).

Ведь нулевого зазора/натяга в принципе быть не может, потому что тогда двум абсолютно гладким поверхностям в области границ пришлось бы соприкоснуться ядрами атомов друг друга?

Или граница тела проводится не по ядрам пограничных атомов, а по внешней орбитали этих атомов?
Или по критическому радиусу отталкивания (слабое/сильное взаимодействие)?
Аноним 28/10/20 Срд 01:15:05 519659505
Есть ли сознание и осознанность у животных, там у собачек, дельфинчиков? Вот если меня как зрителя через нейроинтерфейс повестить в голову тигра, что какие когнитивные процессы я смогу там наблюдать?
Аноним 28/10/20 Срд 01:17:53 519660506
image.png 626Кб, 607x381
607x381
>>519659
Зависит от определения понятий "сознание" и "осознанность".
Аноним 28/10/20 Срд 01:59:33 519661507
>>519657
В пределах некоторой точности, граница тела проходит по орбиталям электронов. Но всякие тела могут всяческие взаимодействовать с друг другом на расстояние сильно прерывающие размер орбитали. Так что все зависит от конкретного случая. В добавок важна геометрия(кривизна) поверхности границы.
Короче "по критическому радиусу отталкивания".
Аноним 28/10/20 Срд 04:24:13 519663508
>>519638
Это самая никчемная попытка выдать свои маняфантазии за доказательство за шесть лет на обосцае.
Аноним 28/10/20 Срд 09:32:36 519666509
>>519651
>оказалось там школьники, наверно я сюда пришел
>/sci/
лол
Аноним 28/10/20 Срд 11:21:06 519669510
>>519666
А куда еще? На бордах нет тематических разделов
Аноним 28/10/20 Срд 11:28:16 519670511
>>519660
Ну как у нас. «О, у меня зачесалась жопа, надо пальчиками пошерудить там... о вот я и почесал жопу» или «так, я немного проголодался, надо бы поймать зебру»
Аноним 28/10/20 Срд 14:17:57 519681512
>>519639>>519640>>519641>>519642
>>519643>>519644>>519663
Такое впечатление, что это один и тот же чел пердачеллом своим бомбит и семенит ITT.
Если ты считаешь, что времени нет как таковго, то поясни-ка лучше, за свой пространственно-световой континуум,
и как через свет у тебя выводится гравитация, в частности - гравитационная постоянная (которая для тебя не фундаментальна, в фундаментальном пространстве-свете)?
Аноним 28/10/20 Срд 14:18:55 519682513
>>519681
А ещё лучше - притащи ещё и пруфы, в виде доказательств, ололо.
Аноним 28/10/20 Срд 14:19:25 519683514
>>519682
Желательно - железобетонных.
Аноним 28/10/20 Срд 15:50:41 519688515
>>519670
А где тут сознание? Простейшие ответные реакции на внешние, или не очень, воздействия.
Аноним 28/10/20 Срд 15:52:33 519689516
>>519681
>Такое впечатление, что это один и тот же чел пердачеллом своим бомбит и семенит ITT.
Бомбишь тут только ты со своим "СУЩЕСТВУЕТ!!!1".

>>519682
>А ещё лучше - притащи ещё и пруфы, в виде доказательств
Давайка сначала ты, что-то кроме "АРЯЯЯ, СУЩЕСТВУУУЕТ!!1"
Аноним 28/10/20 Срд 17:25:59 519693517
>>517798 (OP)
Подвезут ли вообще general intelligence ИИ который будет превосходить человеков, или обезьяниь мозги так и останутся вершиной интеллектуальной эволюции?
Смогут ли современные нейросети в перспективе автоматизировать 90% прикладных задач, или это всё наёб для гоев?
Аноним 28/10/20 Срд 18:36:26 519698518
>>519681
>ита всьо симьон
О, точно клиент из дурки.
Аноним 28/10/20 Срд 18:37:20 519699519
>>519689
Я не спроста делают акцент внимания на слове существует. Как что-либо может СУЩЕСТВОВАТЬ, блядь, если времени нет?
Само слово СУЩЕСТВОВАНИЕ подразумевает наличие времени.
Когда ты спрашиваешь СУЩЕСТВУЕТ ЛИ ЧТО-ЛИБО, ты подразумеваешь СУЩЕСТВОВАНИЕ этого чего-либо, в течении некоего времени.
И если перефразировать вопрос "существует ли это", на "существует ли это в течении некоего интервала времени", то эта взаимосвязь ещё более очевидна.

Об этом я и пишу, блядь, уже который пост,
а ты просто семенишь, пердаком бомбишь, и пукаешь что-то невнятное, вроде:
"ко-ко-ко, это маняфантазии", термин "существование" со временем типа не связан,
времени нет, есть волны, есть "пространство-свет",
докажи что СУЩЕСТВУЕТ, блядь, я спрашую а ты доказывай. Лол.

Короче, ИМХО, слово СУЩЕСТВУЕТ, применительно ко времени - некорректно, и некорректно доказывать то, что время СУЩЕСТВУЕТ.
В течении какого времени, существует время, блядь, а? А существует ли то время? А в течении какого времени? Получается рекурсия. И иди нахуй с ней. Время просто ЕСТЬ, оно не СУЩЕСТВУЕТ. Понимэ?
Аноним 28/10/20 Срд 18:52:39 519700520
>>519693
Нахуя тебе? Будешь уран копать руками за миску похлебки.
Аноним 28/10/20 Срд 18:56:00 519701521
>>519700
На вопрос отвечай, еблан. Нахуя оно мне не твоего ума дело.
Аноним 28/10/20 Срд 18:57:10 519702522
>>519651
Начинай с основ:
А. Ленинджер "Основы биохимии"
Все три тома. Это ебать тебя коромыслом библия. Охват - закачаешься. Твоя библиотека без этого - хуйня.
Если сложно - откатись на уровень назад:
Р. Моррисон, Р. Бойд "Органическая химия"
Если вообще нулевой, начинай с нуля:
Г. Сиборг "Химия. Курс для средней школы"
Но это пизда, хуй где найдёшь. Я только в каталоге очень крупной библиотеки видел.
Аноним 28/10/20 Срд 18:59:26 519703523
>>519693
Тут такое дело. Человек до сих пор не разобрался со своим разумом и интеллектом, а ты хочешь что-то запредельного.
Кроме того человеческий разум еще подразумевает социальный аспект. Для того чтобы человек воспринимал тостер разумным, ему нужно этот тостер наделить человеческим поведением, иначе возникает конфуз.
Если отбросить социальный аспект, то главная фишка человека как интеллектуальной системы его возможность обобщать и абстрагировать задачи. Это позволяет ему комбинировать уже готовые выученные решения для новой незнакомой задачи.
Вторая фишка это сознание. Лютая заноса в жопе у философов. Если смотреть внешнее, то сознание это способность систем абстрагировать и обобщать самих себя. И тут возникает проблема в нашем видение разума, потому что мы не постигли само сознание.
И чтоб построить йоба ИИ, нам нужно создать машину с сознанием с нуля. Пока что это невозможная задача.

Не стоит питать накладывать на нейросети большие надежды, они проигрывают во многих задачах многим алгоритмам. А сложные нейросети мы еще толком не научились делать и похоже они все равно оказываются избыточными по ресурсам.
Скорее будущее будет за гибридами или вообще новой концепции интеллектуальных систем. И может быть уже завезут какой-нибудь скайнет, который поработит отупевших человеков.

И уже современные машины автоматизировали огромное количество рутинных задач, так что может быть ты дождешься автоматического подтирателя задницы.
Аноним 28/10/20 Срд 19:04:11 519704524
>>519701
У у бля так и представил такого быдларя стоящего перед Альфой, Убермайнодом, Хайвом, Жнецами и Браниаком и агрессивно стучащего по терминалу ЫЫЫЕ ЕПТА БЛЯ СЧИТАЙТЕ ДАВАЙ СЛЫШ ЭЭЭ БЛЯ.
Аноним 28/10/20 Срд 19:18:07 519705525
>>519703
>главная фишка человека как интеллектуальной системы его возможность обобщать и абстрагировать задачи
Ну если кучка нейронов может это, то чому кучка транзисторов не може?
>И чтоб построить йоба ИИ, нам нужно создать машину с сознанием с нуля
Человек не обладал сознанием с нуля, он эволюционировал же.
>современные машины автоматизировали огромное количество рутинных задач
Производители не торопятся пилить lights-out заводы. Да и вон Машк хотел фуллавто фабрику по производству Тесл запилить, обосрался.
Аноним 28/10/20 Срд 19:39:54 519709526
>>519702
Cпасибо, а химию тоже так понимаю надо на таком фундаментальном уровне знать?
Аноним 28/10/20 Срд 19:46:09 519710527
>>519705
>Ну если кучка нейронов может это, то чому кучка транзисторов не може?
Живой мозг это не просто куча статичных нейронов. Нейроны имеют внушительную дерево из дендритов(тавтология, лол), которые непрерывно растут и отмирают. Даже обычный синапс это по сути если одноразовая, то ограниченная по пользованию структура. Если не брать периферийную нервную систему, где все более менее одно и тоже, в мозгу у каждого нейрона есть набор одноразовых специфических рецепторов. Эти рецепторы чувствительны на определенные вещества. Из-за это в мозгу есть химические каналы связи помимо синапса.Тащемта эти каналы связи эволюционно первые, но это не отменяет их важность.
Транзисторы статическая структура, и пока что не созданы технологии, которые могли динамически переключать соединения и имеет альтернативные сигнальные каналы. Мы даже из 2D топологии выйти не можем. А попытки симуляции не выдерживают масштабы: для симуляции десяток тысяч нейронов требует кристалл, который требует на порядок больше энергии и размерами больше чем объект симуляции.
Так что на данном уровне знаний и технологии нет.
>Человек не обладал сознанием с нуля, он эволюционировал же.
Это потребовало миллионы лет эволюции. А создание современных систем в чем-то тоже похожи на эволюцию, которая затратна по временем. Так что повторить путь матушки природы нам потребуется хоть и меньше, но все равно очень много лет. Впрочем это не отменяет того, что человек получить некие качественно новые знания.
>Производители не торопятся пилить lights-out заводы. Да и вон Машк хотел фуллавто фабрику по производству Тесл запилить, обосрался.
Политота, тут вообще нельзя что-то уверенно говорить.
Аноним 28/10/20 Срд 19:56:31 519713528
>>519703
>Вторая фишка это сознание. Лютая заноса в жопе у философов.
Его нет. Ты в говне.
Метцингер
Аноним 28/10/20 Срд 19:59:43 519714529
>>519710
>А попытки симуляции не выдерживают масштабы: для симуляции десяток тысяч нейронов требует кристалл, который требует на порядок больше энергии и размерами больше чем объект симуляции.
Т.е. вопрос в мощностях и софте?
Аноним 28/10/20 Срд 20:12:25 519716530
>>519714
>Т.е. вопрос в мощностях и софте?
Скорее концепции и подходу к созданию вычислительных систем.
Аноним 28/10/20 Срд 20:13:51 519718531
Аноним 28/10/20 Срд 20:35:20 519719532
>>519718
Сейчас простая концепция.
Есть процессор выполняет элементарные операции по программе. Такой формат очень очень удобен и легко маштабируется, в первую очередь в физически на кристалле.
Даже в распределенных и распараллельных системах как видеокарты те же элементарные процессоры.
Для эффективной симуляции нейросетей нам придется создать супер параллельный процессор. Раньше пытались делать такие, но уперлись к определенным трудностям.
Первая это поверхностная топология кристалла, транзисторы распределяются по поверхности, что накладывают определенные побочные издержки. Конечно можно поиграться с 3Д структурой, но это за пределами текущих технологий.
Вторая это задачи и алгоритмы которые мы накладываем на процессоры. Эффективность параллельных систем при увлечение масштабов не растет для многих задач и алгоритмов.
Третье. Алгоритмы до сих пор тащат. Нейросети сейчас не более чем хайповая игрушка.
Аноним 28/10/20 Срд 20:54:16 519720533
>>519719
Благодарю за пояснения
Аноним 28/10/20 Срд 20:54:41 519721534
>>519699
>Я не спроста делают акцент внимания на слове существует.
Конечно, ведь ты шизик и это твоя маняфантазия, логично что ты только это и можешь раз за разом шизоидно повторять.

Вот как опять.
Аноним 28/10/20 Срд 22:03:38 519728535
Аноним 28/10/20 Срд 23:30:08 519731536
>>519709
Очень, очень неплохо бы понимать. Ну не то чтобы как минимум докторскую на полочку положить, а иначе нахуй, но хорошее такое представление окажется невероятно полезным. По крайней мере от того, что ты не можешь осилить свежую интересную статью, только потому что не очень понимаешь всё это химическое колдунство, - будет сильно бомбить.
Аноним 30/10/20 Птн 00:38:57 519814537
>>519688
Я не просто про реакции, а осознанное рефлексирование этих воздействий.
Аноним 30/10/20 Птн 00:59:14 519821538
>>518170
>Потому что эгоизм и жадность, это свойства человека, которые нельзя отнимать. Если их отнять, будет уже не человек нахуй.
Воровать у людей и убивать людей - это свойства человека. Если их отнять, будет уже не человек нахуй.
Аноним 30/10/20 Птн 01:09:30 519822539
>>519821
Путаешь желания и действия. Без действий ты всё еще человек. А вот без желаний уже нет.
Аноним 30/10/20 Птн 18:07:56 519876540
откуда мы знаем что вселенной 13.8млдр лет?
Аноним 03/11/20 Втр 18:38:26 520240541
13472845403100.png 894Кб, 599x899
599x899
>>517971
Hу ты и дoлбoёб, xуле. Я вoт за 45к pабoтаю, не напpягаяcь, на еду макcимум 6к в меcяц уxoдит, пpи тoм чтo я и мяco ем и вcё ocтальнoе. Ещё на 5к уxoдит на бенз и 7к на xату. Итoгo, oкoлo 20к в меcяц на жизнь + 25к я инвеcтиpую вo вcякoе, пoлучая дoпдoxoд. Mне нopм, для муxocpанcка даже заебиcь.
Ho ктo же винoват, OП, чтo ты такoй тупoй еблан, чтo cпocoбен тoлькo на pабcкий неквалифициpoванный тpуд за гpoши? Toлькo ты. Hадo былo не дpoчить и cидеть на дваче, а ocваивать пpoфеccии. Ho cудя пo тoму, чтo ты какoй-тo xуйни на 450 pублей напoкупал, вмеcтo нopмальнoй еды, ты pеальнo не ocoбo умный.
Аноним 12/11/20 Чтв 08:16:32 520869542
>>519876
по разным наблюдения света от дальних галактик и расчётам. Но на самом деле точность +- 1 млрд.
Настройки X
Ответить в тред X
15000
Макс объем: 40Mб, макс кол-во файлов: 4
Кликни/брось файл/ctrl-v
Стикеры X
Избранное / Топ тредов